Вы находитесь на странице: 1из 64

See discussions, stats, and author profiles for this publication at: https://www.researchgate.

net/publication/326460294

Atomic Structure & Basic Concepts of Chemistry

Chapter · April 2018

CITATIONS READS
0 19,354

1 author:

Ramesh Duraisamy
Arba Minch University
20 PUBLICATIONS   121 CITATIONS   

SEE PROFILE

Some of the authors of this publication are also working on these related projects:

Phytochemical Screening, Toxicological Profile and In-Vivo Anti-Arthritic Activity of “Cymbopogon citratus” Leave Extracts in Lab Animals. View project

Fish Leather processing and sugar technolgy researches View project

All content following this page was uploaded by Ramesh Duraisamy on 18 July 2018.

The user has requested enhancement of the downloaded file.


E – MATERIAL for CHEMISTRY GRADUATES

Atomic Structure & Basic Concepts of Chemistry

Prepared By

Dr. RAMESH DURAISAMY


Assistant Professor
Department of Chemistry
Arba Minch University
Arba Minch (Ethiopia)
Table of Contents

Chapter. I Introduction to Chemistry ..............4 2.4.2 Characteristic of electromagnetic


1.1 Suggestions for Studying General Chemistry, radiation ....................................................... 15
practicing with the new fact or skill. ...................4 2.4.3 Properties of Waves and Electromagnetic
1.1.1 Take responsibility for your own learning radiation ....................................................... 15
.......................................................................4 2.4.4 Characteristics of wave motion............. 15
1.1.2 Studying .................................................4 2.5 Electromagnetic Radiation .......................... 16
1.1.3 Develop an effective set of study skills ...4 2.6 Quantum effects and photons ...................... 16
1.1.4 Develop your problem-solving skills ......4 2.6.1 Planck’s Quantization Energy .............. 17
1.2 The Scientific Methods .................................5 2.6.2 Photoelectric effect .............................. 17
1.2.1 Law and Hypothesis ...............................5 2.7 Spectrum .................................................... 17
1.2.2 Theory....................................................5 2.7.1 Atomic Spectra .................................... 17
1.3 Measurement and units in chemistry .............5 2.7.2 Emission Spectra.................................. 17
1.3.1 SI units ...................................................5 2.7.3 Absorption Spectra............................... 18
1.3.1.2 Density and Specific gravity ................6 2.8 Bohr model of the atom .............................. 19
1.3.1.4 Mole concept...........................................7 2.8.1 Limitations of Bohr’s Theory: .............. 20
1.4 The Classification of Matter ..........................7 2.9 Dual Nature of the Electron ........................ 20
1.4.1 Three states of matter .............................7 2.10 Quantum mechanics .................................. 20
1.5 Some Basic Definitions. ................................8 2.10.1 Heisenberg Uncertainty Principle ....... 20
1.5.1 Physical and Chemical properties ...........8 2.10.2 Schrodinger wave mechanics ............. 21
1.5.2 Atoms and Molecules .............................8 2.11 The Quantum mechanical Description of the
1.5.3 Substances and Mixtures ........................9 hydrogen atom .................................................. 21
1.5.4 Elements and Compounds ......................9 2.12 Quantum Numbers .................................... 21
1.5.5 Molecules and Ions............................... 10 2.12.1 Principal quantum number (n) ............ 22
1.5.5.1 Molecules.............................................. 10 2.12.3 Magnetic quantum number ................. 22
1.6 Significant Figures ...................................... 10 2.12.4 Spin quantum number ........................ 23
1.6.1 Guidelines for using Significant figures 10 2.13 Atomic Orbitals ........................................ 23
2.13.1 The energies of Orbitals ..................... 23
Chapter-II Atomic Structure....................... 12 2.13.2 Electron Configuration ....................... 24
2.1 Atomic theory & fundamental particles ....... 12 2.13.3 General Rules for Assigning Electrons
Atomic Theory.................................................. 12 to Atomic Orbitals ........................................ 24
2.2 The structure of the Atom ........................... 12 2.13.4 Method of writing electronic
2.2.1 The Electron ......................................... 12 configuration: ............................................... 24
2.2.2 The proton and Nucleus........................ 13 nlx method and Box method .......................... 24
2.2.3 The Neutron ......................................... 14 2.14 Pauli Exclusion Principle .......................... 25
2.3 Atomic number, Mass number and Isotopes 14 2.15 Hund’s Rule .............................................. 25
2.4 Nature of light and Electromagnetic radiation 2.15.1 Paramagnetism and Diamagnatism ..... 25
......................................................................... 14 2.15.2 The shielding effect in Many-Electron
2.4.1 Corpuscular Theory .............................. 14 Atoms ........................................................... 25
2.16 The Building-Up Principle ........................ 26
Chapter-III Periodic Properties ............. 28 5.7 Precipitation Reactions ............................... 53
3.1 Need to classify elements ............................ 28 5.7.1 Solubility ............................................. 53
3.2 Brief history of modern periodic law and 5.8 Molecular equations and Ionic equations..... 55
modern periodic table.................................. 28 5.9 Acid-base neutralization.............................. 57
3.2.1 Modern periodic Law ........................... 28 5.10 Oxidation-Reduction Reactions ................. 57
3.2.2 Modern periodic table ( Long form of... 28 5.10.1 Oxidation Number.............................. 58
periodic table) ............................................... 28 5.10.2 Types of redox reactions .................... 60
3.3 Class of periodic elements : Based on 5.10.3 Balancing of Redox reactions ............. 62
electronic configuration .................................... 29
3.3.1 s - block elements ................................. 30
3.4 Chemical Properties of s-block elements ..... 32
3.4.1 IA Group elements (Alkali metals) ....... 32
3.5 p-block elements ......................................... 33
3.5.1 Introduction.......................................... 33
3.5.2 General aspects and trends in the p-block
elements........................................................ 33
3.6 Groups in p-block elements ......................... 34
3.6.1 Boron Family ....................................... 35
3.6.2 Elements of group IV A (Carbon family)
..................................................................... 36
3.6.3 Elements of VA (15th) group (Nitrogen
family) .......................................................... 37
3.6.4 Oxygen family (Elements of group VIA)
..................................................................... 38
3.7 IIA Group elements (Alkaline earth metals)
......................................................................... 39

Chapter - IV Molecular Geometry ........ 40


4.1 Molecular Geometry ................................... 40
4.2 Dipole moments .......................................... 42
4.3 Valence Bond Theory ................................. 43

Chapter. V Chemical Reactions .................. 44


5.1 Chemical Equations .................................... 44
5.1.1 Writing chemical equations .................. 44
5.2 The mole concept ........................................ 44
5.3 Balancing of the chemical equations ........... 46
5.4 Amount of reactants and products ............... 47
5.5 Limiting Reagents ....................................... 50
5.6 Reaction Yield ............................................ 51

Atomic Structure & Basics of Chemistry Page 3


Chapter. I Introduction to Chemistry  Take notes and review them immediately
after class.
1.1 Suggestions for Studying General Chemistry  Use the five R's of note taking
One of the most common challenges that General 1. Record: The meaningful ideas and
Chemistry students have is that they don't realize the concepts
importance of being actively involved in their own 2. Reduce: After class, summarize the main
learning. Many seem to think that coming to class ideas and concepts
and passively listening to the instructor's pearls of 3. Recite: Say out loud in your own words
the main ideas of the class.
wisdom is all that is needed. Generally, everyone is
4. Reflect: Take a few minutes to ponder
a little different; everyone has at least slightly
over the main ideas of the class.
different intellectual equipment and a different set of
5. Review: Once a week, review the ideas of
life and school experiences, which they bring to the
all lectures.
learning situation. Therefore, the learning
experience is a little different for each person, even 1.1.2 Studying
those in same classroom. The person who wishes to  Study before each class. The material will be
become a learner must be actively involved in fresh in your mind.
constructing his or her own set of ideas. This  Allow time for sleep.
process will be different in its details for each  Set realistic goals for yourself. You can reward
yourself for being successful. If you waste all
person, but each must go through a similar process
afternoon and then set a goal of studying
of hearing (or seeing or reading) the new item,
chemistry from 6:00 p.m. to midnight, you may
comparing it with what is already in his or her mind,
not be realistic.
and integrating the new information into the
memory bank. This is not a simple process--it 1.1.3 Develop an effective set of study skills
requires work! The work may take the form of  Pace your studying throughout the week and
reading and re-reading, thinking about the new throughout the semester
information and how it fits in, practicing with the  Get enough rest and otherwise take care of your
new fact or skill. health.
1.1.1 Take responsibility for your own learning  Find a place in which you can study effectively
 Put in the required effort  Form a study group with classmates and discuss
 Read the assignments carefully before words, concepts, problem-solving strategies, etc.
coming to class, and then again after class. to the benefit of all the members of the group.
 Do the assigned problems and may be
extras for good measures. 1.1.4 Develop your problem-solving skills
 Come to class prepared and on time. Class  Sharpen your basic mathematical skills
attendance is important even if your  Learn how to use your calculator effectively.
instructor doesn't specifically require it.  Learn & practice using units for physical
 Take complete notes in class quantities.
 Pay attention in class. Listen to your  If you need a piece of information, look it up
instructor, not to the student next to you. using the index of your text or other suitable
 If you miss class, make sure that you get all source. Don't be afraid to be wrong, as Sir Isaac
the necessary information from the Newton said many years ago, "Truth comes more
instructor or another responsible student. easily out of error than out of confusion."
 Try to capture the ideas & concepts of lecture
Atomic Structure & Basics of Chemistry Page 4
1.2 The Scientific Methods be determined directly. Microscopic properties, on
The study of chemistry concerned with the the atomic or molecular scale, must be determined
observations, theories and laws that give this science by an indirect method. A measured quantity is
its foundation and that form a framework into which usually written as a number with an appropriate unit.
information fits to make an integrated area of The same is true in chemistry; units are essential to
knowledge. This framework develops from the stating measurements correctly.
persuit of answers to many questions, each of which
can be subjected to experimental investigation by an 1.3.1 SI units
approach often called the scientific method. Many years’ scientists recorded measurements in
1.2.1 Law and Hypothesis metric units, which are related decimally (i.e. by
Scientists identify problems or questions through powers of 10). On 1960‘s the general conference of
their own observations and experiments and get the weights and measures notified, the international
conclusions, compare with earlier observations of system of units (SI). The SI system consists of the
the same. The first step of the scientific method, to a following seven units, from which other units of
problem involves carefully planning the experiments weight and measure can be derived. Measurements
to gather the information about all phases of the that we will utilize frequently in our study of
problem. The results are examined for general chemistry include time, mass, volume, density and
relationships that will unify the observations. temperature.
Sometimes a wide variety of observations can be
summarized in a general verbal statement or SI base units:
mathematical equation known as a law. ---------------------------------------------------------------
One example is the law of conservation of mass, Physical property Name of unit Symbol
which summarizes the results of thousands of ---------------------------------------------------------------
experimental observations. More often, is suggested Length meter m
Mass kilogram kg
a tentative explanation. Such a proposal is called a
Time second s
hypothesis.
Electric current Ampere A
1.2.2 Theory
Temperature Kelvin K
A hypothesis is tested by further experiments, and, if Luminous intensity Candela cd
it is capable of explaining the large body of Amount of substance mole mol
experimental data, it is dignified by the name of ---------------------------------------------------------------
theory. The data obtained in a research study may be the powers to which 10 is raised as, common
both quantitative, consisting of general observations prefixes used in the metric system
about the system, and qualitative, comprising ---------------------------------------------------------------
numbers obtained by various measurements and Prefix Symbol Factor Example
observations. ---------------------------------------------------------------
1.3 Measurement and units in chemistry pico p 10-12 1 picoeter (pm)
-9
nano n 10 1nano gram (ng)
The measurements chemists make are often used in
micro  10-6 1microliter (L)
calculations to obtain other related quantities.
milli m 10-3 2milliseconds(ms)
Different instruments enable us to measure a centi c 10 -2
5centimeters(cm)
substance’s properties. The meter stick measures deci d 10-1 1 deciliter (dL)
length or scale; the buret, the pipette, the graduated 3
kilo k 10 1kilometer(km)
cylinder and volumetric flask measure volume; the mega M 106 3 mega grams (Mg)
9
balance measures mass; the thermometer measures giga G 10 5 giga meters (Gm)
temperature. These instruments provide tera T 1012 1 tera litre (TL)
measurements of microscopic properties, which can ---------------------------------------------------------------

Atomic Structure & Basics of Chemistry Page 5


Chemists are interested primarily in mass, which can 1 g/cm3 = 1g/mL = 1000 kg/m3
be determined readily with a balance; the process of 1g/L = 0.001g/mL
measuring mass is called weighing. The SI base unit The term specific gravity denotes the ratio of the
of mass is the kilogram (kg), but in chemistry the density of a substance to the density of a reference
smaller gram (g) is more convenient; substance. Usually, the reference substance for
1 kg = 1000g = 1x103g solids and liquids is water.
1.3.1.1 Mass and Weight Density of substance
Mass is a measure of the amount of matter in an Specific gravity = -----------------------------------------
object, it is an invariable quantity. On the other Density of reference substance
hand, the weight of a body is the force that gravity Specific gravity, being the ratio of two densities, has
exerts on the body; it is variable, since the attraction no units. Usually the specific gravity of any
depends on the distance from planet’s center. substances is numerical.
Scientists measure quantities of matter in terms of Workout.1 Calculate the density and specific
mass rather than weight because the mass of a body
gravity of a body that has a mass of 321g and a
remains constant, whereas its weight is an accident
volume of that 45 cm3 at 250C.
of its environment. However, that the term weight is
often used loosely for the mass of a substance.
Mass 321g
The mass of an object is found by comparing it to Density = --------------- = ------------ = 7.13 g/cm3
other objects of known mass. The instrument used to Volume 45 cm3
make this comparison is called a balance. The
standard object to which all SI and other metric units Density of sample
Specific gravity = --------------------------
of mass are referred a cylinder of platinum-iridium
Density of water
alloy. The unit of mass, the kilogram (kg), is defined The density of water at 250C is 0.99 g/cm3, specific
as the mass of this cylinder; some times kilogram gravity of sample is 7.13.
expressed in pounds (1 kilogram is about 2.2
pounds). The gram (g) is equal to exactly 0.001 Workout.2 What is the mass in kg of 10.5g (39.7L)
times mass of the std kilogram (1x10-3 kg) and is of gasoline with a specific gravity of
nearly equal to the weight of 1cm3 of water. 0.82?
1.3.1.2 Density and Specific gravity Because gasoline has a specific gravity of 0.82 the
One of the physical properties of a solid, liquid or a density of 1mL of gasoline is 0.82 times that of
gas is its density. Density is defined as mass per unit 1mL of water.
volume. This is mathematically expressed as, Density = Specific gravity x density of water
Mass M = 0.82 x 0.997 g/cm3
Density = ------------- (or) D -------
Volume V = 0.82 g/cm3 (or 0.82 g/mL)
Conversion of 39.7 L to mL followed by
Substances usually have different densities, multiplication by the mass of 1mL of gasoline gives
Substances Density (at 250C and 1atm.) the total mass.
Air 1.29 g/L
He 0.179 g/L
H2O 0.997 g/cm3 1000 mL
Glycerin 1.26 g/cm3 Volume = 39.7L x ------------- = 39700 mL
Hg 13.6 g/cm3 1L
Salt 2.17 g/cm3
Iron 7.86 g/cm3 Mass = Density x Volume
Silver 10.5 g/cm3 = 0.82 g/mL x 39700 mL
= 33,000 g
Here, density for all cases expressed as, g/cm3 Mass = 33kg

Atomic Structure & Basics of Chemistry Page 6


Workout.3 A piece of gold in got with a mass of carbon atom. Because we have 3x1022 atoms of
301g has a volume of 15.6 cm3. Calculate carbon we can make,
the density of gold. 1 CCl4 molecule
D = M/V = 301g / 15.6 cm3 22
3 x 10 C atoms x -------------------------
D = 19.3 g/cm3 1 C atom
22
1.3.1.3 Volume = 3x10 CCl4 molecules
The SI unit of length is the meter (m) and the SI
derived unit for volume is the cubic meter (m3). Problem.5 How many moles of chlorine atom are
Generally, chemists work with smaller volumes, required to react with 0.050 mol of
such as cubic centimeter (cm3) and the cubic carbon atoms to convert all of the carbon atoms to
decimeter (dm3). CCl4.
1cm3 = (1x10-2m)3 = 1x10-6m3 The formula CCl4 indicates that there are 4 Cl atoms
1dm3 = (1x10-1m)3 = 1x10-3m3 for each C atom in a molecule, thus we must take
Another common unit of volume is the liter (L). A four times as many Cl atoms as C atoms,
liter is the volume occupied by one cubic decimeter. 6.023x1023 C atoms
One liter volume is equal to 1000 mL or 1000 cm3. 0.050 mol C atoms x -------------------------------
1L = 1000 mL 1 mol C atoms
= 1000 cm3 (1mL = 1cm3) 22
= 3x10 C atom
= 1 dm3
1.3.1.4 Mole concept 4 Cl atoms
Usually, number of atoms is measured in the units of 3x10 C atoms x ---------------- = 1.2x1023 Cl atoms
22

6.023x1023. This unit called one mole of atoms 1 C atoms


(1mol). The number of atoms in one mole of any calculate the number of moles of Cl atoms from the
element (1 mol. of atom has 6.023 x 1023 atoms) is definition of a mole, 1 mole of Cl atoms contains
also called Avogadro’s number. For example, let us 6.023x1023 Cl atoms [1 mol Cl atom / 6.023x1023 Cl
consider a sample of carbon tetra chloride by atoms)
combining 1 mole of carbon atoms with 4 moles of 1 mol Cl atom
Cl atoms. This would be a produce 6.023 x 1023 23
1.2x10 Cl atoms x -----------------------------
molecule of CCl4. 6.023x1023 Cl atoms
Workout.4 How many CCl4 molecules can be made = 0.20 mol of Cl atom
by the combination of 0.050 mol of 1.4 The Classification of Matter
carbon atoms with Cl atom? The chemistry deals the study of matter and the
The formula of CCl4 tells us that one CCl4 molecule changes it undergoes. Matter is anything that
contains one carbon atom (1 molecule CCl4/ 1 atom occupies space and has mass. Thus, everything in
of C). If we can determine number of carbon atoms the universe has a “chemical” connection. Chemical
present, we can determine the number of CCl4 distinguish among several subcategories of matter
molecules that can be made. One mole of C atoms, based on composition and properties. The
contains 6.023x1023 C atoms; 0.050 mole of C classification of matter includes substances,
atoms contains, mixtures, elements and compounds, as well as atoms
0.050 mol C atoms x 6.023 x 1023 mol of C atoms and molecules.
----------------------------------------------------------------- 1.4.1 Three states of matter
1 mol of C atoms All substances, at least in principle, can exist in
= 3x1022 C atoms three states: solid, liquid, and gas. The gases differ
A single CCl4 molecule contains 1 atom of carbon.
from liquids and solids in the distances between the
Therefore, we can make 1 molecule of CCl4 for each
molecules. In a solid, molecules are held close

Atomic Structure & Basics of Chemistry Page 7


together in an orderly fashion with little freedom of All measurable properties of matter fall into one of
motion. Molecules in a liquid are close together but two additional categories: extensive properties and
are not held so rigidly in position and can move past intensive properties. The measured value of an
one another. In a gas, distances that are large extensive property depends on how much matter is
compared with the size of the molecules separate the being considered. Mass, which is the quantity of
molecules. matter in a given sample of a substance, is an
The three states of matter can be interconverted extensive property. More matter means more mass.
without changing the composition of the substance. Values of the same extensive property can be added
Upon heating, a solid (ice) will melt to form a liquid together. For example, two copper pennies will have
(water) the temperature at which this transition a combined mass that is the sum of the masses of
occurs is called the melting point. Further heating each penny, and the length of two tennis courts is
will convert the liquid into a gas, this conversion the sum of the lengths of each tennis court. Volume,
take place at the boiling point of the liquid. On the defined, as length of cubed, is another extensive
other hand, cooling a gas will cause it to condense property. The value of an extensive quantity
into a liquid. When the liquid is cooled further, it depends on the amount of matter.
will freeze into the solid form. The measured value of an intensive property does
1.5 Some Basic Definitions: Mass and Weight, not depend on how much matter is being considered.
Physical and Chemical properties, Atoms and Density, defined as the mass of an object divided by
molecules, Substances and Mixtures, Elements its volume, is an intensive property. We consider the
and Compounds. temperature. Suppose that we have two beakers of
1.5.1 Physical and Chemical properties water at the same temperature. If we combine them
Substances are identified by their properties as well to make a single quantity of water in a larger beaker,
as composition. Color, melting point, and boiling the temperature of the larger quantity of water will
point are physical properties. A physical property be the same as it was in two separate beakers.
can be measured and observed without changing the Unlike mass, length, and volume, temperature and
composition or identity of a substance. For other intensive properties are not in additive.
1.5.2 Atoms and Molecules
example, we can measure the melting point of ice by
An atom is the smallest particle of an element that
heating a block of ice and recording the temperature
can enter into a chemical combination. The word
at which the ice is converted to water. Water differs
atom comes from the Greek word atomos, which
from ice only in appearance, not in composition, so
means indivisible. John Dalton, an English chemist
this is a physical change; we can freeze the water to
and physicist, presented an atomic theory and
recover the original ice. Therefore, the melting point
supported with quantitative measurements, found it.
of a substance is a physical property. Similarly,
A molecule is the smallest particle of an element or
when we say that helium gas is lighter than air, we
compound that can have a stable, independent
are referring to a physical property.
existence. A molecule may consist of a single atom,
On the other hand the statement. “Hydrogen gas
as in helium, of two or more identical atoms, as in
burns in oxygen gas to form water”, describes a
nitrogen and sulfur, or of two or more different
chemical property of hydrogen, because in order to
atoms, as in water. Water has a definite composition
observe this property we must carry out a chemical
and a set of chemical properties that enable us to
change, in this case burning. After the change, the
recognize it as a distinct substance. Each water
original chemical substance, the hydrogen gas, will
molecule is a unit that contains two hydrogen atoms
have vanished, and all that will be left is a different
and one oxygen atom. Subdivision of a water
chemical substance. In the case, of water we cannot
molecule results in the formation of the gases
recover the hydrogen from the water by means of a
hydrogen and oxygen, each of which has properties
physical change, such as boiling or freezing.

Atomic Structure & Basics of Chemistry Page 8


quite different from those of water and those of each For convenience, chemists use symbols of one or
other. two letters to represent the elements. The first letter
1.5.3 Substances and Mixtures of a symbol is always capitalized, but any following
A substance is a form of matter that has a definite letters are not. For example, Co is the symbol for the
(constant) composition and distinct properties. element cobalt, whereas CO is the formula for the
Examples are water, ammonia, sucrose, gold and carbon monoxide molecule. Names and symbols of
oxygen. Substances differ from one another in some of the more common elements are presented
composition and can be identified by their as,
appearance, smell, taste, and other properties. -----------------------------------------------------------------
A mixture is a combination of two or more Name Symbol Name Symbol
substances in which the substances retain their -----------------------------------------------------------------
distinct identities. Some familiar examples are air, Aluminum Al Fluorine F
soft drinks, milk, and cement. Mixtures do not have Oxygen O Arsenic As
constant composition. Gold Au Phosphorus P
Mixtures are either homogeneous or heterogeneous. Barium Ba Hydrogen H
When a spoonful of sugar dissolves in water we Platinum Pt Bismuth Bi
obtain a homogeneous mixture in which the Iodine I Potassium K
composition of the mixture is the same throughout. Bromine Br Iron Fe
If sand is mixed with iron filings, however, the sand Silicon Si Calcium Ca
grains and the iron filings remain separate. This type Lead Pb Silver Ag
of mixture is called a heterogeneous mixture Carbon C Magnesium Mg
because the composition is not uniform. Sodium Na Chlorine Cl
Any mixture, whether homogeneous or Manganese Mn Sulfur S
heterogeneous, can be created and then separated by Chromium Cr Mercury Hg
physical means into pure components without
Tin Sn Cobalt Co
changing the identities of the components. Thus,
Nickel Ni Tungsten W
sugar can be recovered from a water solution by
Copper Cu Nitrogen N
heating the solution and evaporating it to dryness.
Zinc Zn
Condensing the vapor will give us back the water
---------------------------------------------------------------
component.
The symbols of some elements are derived from
1.5.4 Elements and Compounds
An element is a substance that cannot be separated their Latin names-for example, Au from aurum
into simpler substances by chemical means. To date, (gold), Fe from ferrum (iron), and Na from natrium
115 elements have been positively identified. (sodium), while most of them come from English
Eighty-three of them occur naturally on Earth. names.
Scientists via nuclear processes have created the Atoms of most elements can interact with one
others. another to form compounds. For example, hydrogen
Only a few elements, such as the gases helium, gas burns in oxygen gas to form water, which has
neon, and argon, consist of a collection of individual properties that are distinctly different from those of
atoms that move about independently of one the starting materials. Water is made up of two parts
another. Other elements, such as the gases nitrogen, hydrogen and one part oxygen. Thus, water is a
oxygen and chlorine, consist of pairs of atoms, each compound, a substance composed of atoms of two
pair moving as a single unit. The element or more elements chemically united in fixed
phosphorous consists of units composed of four proportions.
phosphorus atoms; sulfur, of units composed of
eight sulfur atoms.
Atomic Structure & Basics of Chemistry Page 9
1.5.5 Molecules and Ions atom (Cl) can gain an electron to become the
Out of all the elements, only the six noble gases (He, chloride ion Cl-
Ne, Ar, Kr,Xe, and Rn) exist in nature as single Cl atom: 17 protons and 17 electrons
atoms. For this reason, they are called monatomic Cl- ion : 17 protons and 18 electrons
(mean a single atom) gases. Most matter is Sodium chloride (NaCl), table salt, is an ionic
composed of molecules or ions formed by atoms. compound because it is formed from cations and
1.5.5.1 Molecules anions. An atom can lose or gain more than one
A molecule is formed from least two atoms in a electron. The ions Na+ and Cl- are called monatomic
definite arrangement held together by chemical ions because they contain only one atom. In
forces (called chemical bonds). A molecule may addition, two or more atoms can combine to form an
contain atoms of the same element or atoms of two ion that has a net positive or net negative charge.
or more different elements joined in a fixed ratio. Examples of ions formed by the loss or gain more
Thus, a molecule is not necessarily a compound, than one electron are Mg2+,Fe3+,S2- and N3-.
which is made up of two or more elements. For Polyatomic ions such as OH- (hydroxide ion), CN-
example hydrogen gas is a pure element, but it (cyanide ion) and NH4+ (ammonium ion) ions are
consists of molecules made up of by two H atoms. containing more than one atom.
On the other hand, water is a molecular compound
that contains hydrogen and oxygen in a ratio of two 1.6 Significant Figures
H atoms and one O atom. Like atoms, molecules are Except when all the numbers involved are integers,
electrically neutral. it is often impossible to obtain the exact value of the
The hydrogen molecule, H2, is called a diatomic quantity under investigation. For this reason, it is
molecule because it contains only two atoms of the important to indicate the margin of error in a
same element. Other elements that normally exist as measurement by clearly indicating the number of
diatomic molecule are nitrogen (N2) and oxygen significant figures, which are the meaningful digits
(O2) as well as F2, Cl2 and I2. A diatomic molecule in a measured or calculated quantity. When
can also contain atoms of different elements. For significant figures are used, the last digit is
example are hydrogen chloride (HCl) and CO. understood to be uncertain.
The vast majority of molecules contain more than 1.6.1 Guidelines for using Significant figures
two atoms. They can be atoms of the same element, We must always be careful in scientific work to
as in ozone (O3), which is made up of three atoms of write the proper number of significant figures. In
oxygen, or they can be combinations of two or more general, it is fairly easy to determine how many
different elements. Molecules containing more than significant figures a number has by following these
two atoms are called polyatomic molecules. Like rules:
ozone, water (H2O) and ammonia (NH3) are  Any digit that is not zero is significant. Thus 815
polyatomic molecules. cm has three significant figures, 1.374 kg has
1.5.5.2 Ions four significant figures, and so on.
An ion is an atom or a group of atoms that has a net  Zeros between nonzero digits are significant.
positive or negative charge. The loss of one or more Thus 608 m contains three significant figures,
electrons from a neutral atom results a cation 30,403 kg contains five significant figures, and so
(positive charge). For example, a sodium atom (Na) on.
can readily lose an electron to become sodium  Zeros to the left of the first nonzero digit are not
cation, which is represented by Na+: significant. Their purpose is to indicate the
Na atom: 11 protons and 11 electrons placement of the decimal point. For example,
Na+ ion : 11 protons and 10 electrons 0.08 L contains one significant figure; 0.000756 g
An anion is an ion whose net charge is negative due contains three significant figures, and so on.
to an increase in the number of electrons. A chlorine
Atomic Structure & Basics of Chemistry Page 10
 If a number is greater than 1, then all the zeros only two digits after the decimal point. If the first
written to the right of the decimal point count as digit following the point of rounding off is equal
significant figures. Thus 2.0mg has two to or greater than 5, we add 1 to the proceeding
significant figures, 40.062 mL has five significant digit. Thus 8.727 rounds off to 8.73 and 0.425
figures, and 3.040 dm has four significant figures. rounds of to 0.43.
If a number is less than 1, then only the zeros that  In multiplication and division, the original
are at the end of the number and the zeros that are number that has the smallest number of
between nonzero digits are are significant. This significant figures determines the number of
means that 0.090 kg has two significant figures, significant figures in the final product or quotient.
0.3005 L has four significant figures, 0.00420 The following examples illustrate this rule:
min. has three significant figures, and so on. 2.8 x 4.5039 = 12.61092 --- round off to 13
 For numbers that do not contain decimal points, 6.85/112.04 = 0.0611388789 round off to 0.0611
the trailing zeros may or may not be significant.  Keep in mind that exact numbers obtained from
Thus 400 cm may have one significant figure (the definitions or by counting numbers of objects can
digit 4), two significant figures (40), or three be considered to have an infinite number of
significant figures (400). We cannot know which significant figures. If an object has a mass of
is correct without ambiguity. In this particular 0.2786g, then the mass of eight such object is,
case, we can express the number 400 as 4x102 for 0.2786 g x 8 = 2.229 g
one significant figure, 4.0x102 for two significant
figures, or 4.00x102 for three significant figures. We do not round off this product to one significant
The following example shows the determination figure, because the number 8 is 8.00000 …… by
of significant figures. definition.
Example.1 Determine the number of significant
figures in the following measurements: (a) 468 cm Similarly, to take the average of the two measured
(b) 5.03 g (c) 0.798 m (d) 0.046 kg (e) lengths 6.64 cm and 6.68 cm, we write
23
1.340x10 atoms (f) 6000ml 6.64 cm + 6.68 cm
Solution: We follow the rules for determining ----------------------- = 6.66 cm
significant figures. (a) Three (b) Three (c) 2
Three (d) Two (e) Four (f) This is an Because the number 2 is 2.00000 …….. by
ambiguous case. The number of significant figures definition.
may be four (7.000x103), three (7.00x103), two
(7.0x103), or one (7x103).

A second set of rules specifies how to handle


significant figures in calculations.
 In addition and subtraction, the answer cannot
have more digits to the right of the decimal point
than either of the original numbers. Consider
these examples.
89.332 + 1.1 = 90.432 -------- round off to 90.4
2.097 – 0.12 = 1.977 --------- round off to 1.98
The rounding off procedure is as follows, to
round off a number at a certain point we simply
drop the digits that follow if the first of them is
less than 5. Thus, 8.723 rounds of 8.72 if we want

Atomic Structure & Basics of Chemistry Page 11


Chapter-II Atomic Structure the right kinds of elements, but specific numbers of
these atoms as well. This idea is an extension of law
published by Joseph Proust (in 1799), a French
2.1 Atomic theory & fundamental particles chemist. Proust’s law of definite proportions states
Atomic Theory that different samples of the same compound always
In fifth century, the Greek philosopher Democritus contain its constituent elements in the same
expressed that all matter consists of very small, proportion by mass.
indivisible particles, named as ‘atomos’ (means Dalton’s 2nd hypothesis supports another important
uncuttable or indivisible), but his idea was not law, the law of multiple proportions. According to
accepted. Experimental evidence from early this law, if two elements can combine to form more
scientific investigations provides support for the than one compound, the masses of one element that
‘atomism’ and gradually gave the modern combine with a fixed mass of the other element are
definitions of elements and compounds. It was in in ratios of small whole numbers. Dalton’s theory
1808, English scientist John Dalton, formulated a explains the laws of multiple proportions, different
precise definition of the indivisible building blocks compounds made up of the same elements differ in
of matter that we call atoms. According to this the number of atoms of each kind that combine. For
Daltons’s summarized the atomic theory as, example, carbon forms two stable compounds with
1. Elements are composed of extremely small oxygen, namely carbon monoxide and carbon
particles called atoms. All atoms of a given dioxide.
element are identical, having the same size, mass Third hypothesis
and chemical properties. The atoms of one It is another way of stating the law of
element are different from the atoms of all other conservation of mass, which is that matter can be
elements. neither created nor destroyed. Since matter is made
2. Compounds are composed of atoms of more than of atoms that are unchanged in a chemical reaction,
one element. In any compound, the ratio of the it follows that mass must be conserved as well.
number of atoms of any two elements present is
either an integer or a simple fraction. 2.2 The structure of the Atom
3. A chemical reaction involves only the separation, On the basis of Dalton’s atomic theory, we
combination or rearrangement. can define an atom as the basic unit of an element
Dalton’s concept of an atom was far more detailed that can enter into chemical combination. Dalton
and specific than Democritus and Dalton’s proposed imagined an atom that was both extremely small and
the three hypotheses are as follows, indivisible. However, a series of investigations,
First hypothesis clearly demonstrated that, atoms actually posses
It states that atoms of one element are internal structures; that is, they are made up of even
different from atoms of all other elements. Dalton smaller particles, which are called subatomic
made no attempt to describe the structure or particles. This research discovered the three such
composition of atoms, he had no idea about, what an particles, named as electrons, protons and neutrons.
atom is really like. But he did realize that the 2.2.1 The Electron
different properties shown by different elements One device used to investigate the atomic structure
such as hydrogen and oxygen can be explained by was a cathode ray tube; it is a glass tube from which
assuming that hydrogen atoms are not the same as most of the air has been evacuated. When the two
oxygen atoms. metal plates are connected to a high-voltage source,
Second hypothesis the negatively charged plate, called the cathode,
The 2nd hypothesis suggests that, in order to emits a ray. The cathode ray is drawn to the
form a certain compound, we need not only atoms of positively charged plate, called anode, where it

Atomic Structure & Basics of Chemistry Page 12


passes through a hole and continues traveling to the emit very unusual rays. This highly energetic
other end of the tube. When the ray strikes the radiation penetrated matter, darkened covered
specially coated surface, it produces a strong photographic plates, and caused a variety of
fluorescence, or bright light. substances to fluoresce. Since a magnet could not
In some experiments two electrically charged plates deflect these rays, they could not contain charged
and a magnet were added to the outside of the particles as cathode rays. Rontgen called them X
cathode ray tube. When the magnetic field is on and rays because their nature was not known.
the electric field is off, the cathode ray strikes point After Rontgen’s discovery, Antoine Becquerel,
A. When only the electric field is on, the ray strikes began to study the fluorescent properties of
point C. When both magnetic and the electric fields substances, he found that exposing thickly wrapped
are off or when they are both on but balanced so that photographic plates to a certain uranium compound
they cancel each other’s influence, the ray strikes caused them to darken, even without the stimulation
point B. According to electromagnetic theory, a of cathode rays. Like X rays, the rays from the
moving charged body behaves like a magnet and can uranium compound were highly energetic and could
interact with electric and magnetic fields through not be deflected by a magnet, but they differ from X
which it passes. Since the cathode ray is attracted by rays because they arose spontaneously. One of
the plate bearing positive charges and repelled by Becquerel’s students, Marie Curie, suggested the
the plate bearing negative charges, it must consist of name radioactivity to describe this spontaneous
negatively charged particles. We know these emission of particles and/or radiation.
negatively charged particles as electrons. Since then, any element emits the radiation as
An English physics, J.J.Thomson, used a cathode spontaneously is said to be radioactive.
ray tube and his knowledge of electromagnetic Three types of rays are produced by the decay, or
theory to determine the ratio of electric charge to the breakdown, of radioactive substances such as
mass of an individual electron. The number he came uranium. Two of the three are deflected by
up, -1.76x108 C/g, where C stands for coulomb, oppositely charged metal plates (shown in figure).
which is the unit of electric charge. Thereafter, in a Alpha () rays consist of positively charged
series of experiments carried out, R.A.Millikan particles, called  particles, and therefore are
succeeded in measuring the charge of the electron deflected by the positively charged plate. Beta ()
with great precision. His work proved that rays, or  particles, are electrons and are deflected
the charge on each electron was exactly the same. by the negatively charged plate. The third type of
In his experiment, he suspended the charged drops radioactive radiation consists of high-energy rays
in air by applying an electric field and followed their called gamma () rays. Like X rays,  rays have no
motions through a microscope. Using his knowledge charge and are not affected by an external field.
of electrostatics, Millikan found the charge of an 2.2.2 The proton and Nucleus
electron to be –1.6022x10-19 C. From these data he By the early 1900s, two features of atoms had
calculated the mass of an electron. become clear: they contain electrons, and they are
Charge electrically neutral. To maintain electric neutrality,
mass of an electron = -------------------
an atom must contain an equal number of positive
Charge/mass
and negative charges. Therefore, Thomson proposed
-1.6022x10-19C that an atom could be thought of as a uniform,
= --------------------- = 9.10 x 10-28 g positive sphere of matter in atoms.
- 1.76 x 108 C/g The New Zealand physicist Ernest Rutherford,
studied with Thomson at Cambridge University,
Radioactivity
decided to use  particles to probe the structure of
The German physicist Wilhelm Rontgen noticed (in
atoms. Rutherford carried out a series of
1895), that cathode rays caused glass and metals to
Atomic Structure & Basics of Chemistry Page 13
experiments using very thin foils of gold and other equal to the number of electrons, so the atomic
metals as targets for  particles from a radioactive number also indicates the number of electrons
source. present in the atom. For example, the atomic number
He was observed that the majority of particles of nitrogen is 7. This means that each neutral
penetrated the foil either un-deflected or with only a nitrogen atom has 7 protons and 7 electrons.
slight deflection. But, a  particle was scattered at a Mass number
large angle. Rutherford, explained the results of the The mass number (A) is the total number of protons
-scattering experiment in terms of a new model for and neutrons present in the nucleus of an atom of an
the atom. According to Rutherford concept, most of element. Except for the most common form of
the atom must be an empty space. This explains why hydrogen, which has one proton and no neutron, all
the majority of  particles passed through the gold atomic nuclei contain both protons and neutrons. In
foil with little or no deflection. The atoms have general the mass number is given by,
positive charges, which is in central core of the Mass number = No. of protons + No. of neutrons
atom, called nucleus. The positively charged = Atomic number + No. of neutrons
particles in the nucleus are called protons. In The number of neutrons in an atom is equal to the
separate experiments, it was found that each proton difference between the mass number and the atomic
carries the same quantity of charge as an electron number, (A-Z). For example, the mass number of
and has a mass of 1.6262 x 10-24 g about 1840 times fluorine is 19 and the atomic number is 9 (indicate 9
the mass of the oppositely charged electron. protons in the nucleus). Thus the number of neutrons
2.2.3 The Neutron in an atom of fluorine is 19-9=10. The atomic
Rutherford’s model of atomic structure left one number, number of neutrons and mass number all
major problem unsolved. It was known that must be positive integer (whole numbers).
hydrogen, the simplest atom, contains only one The accepted way to denote the atomic number and
proton and that the helium atom contains two mass number of an atom of an element is
protons. Therefore, the ratio of the mass of a helium represented as ZXA, A is mass number and Z is an
atom to that of a hydrogen atom should be 2:1. In atomic number.
reality, however, the ratio is 4:1. Rutherford and Isotopes
others postulated that there must be another type of Most of the elements have two or more isotopes;
subatomic particle in the atomic nucleus; another atoms that have the same atomic number but
English physicist, James Chadwick, provided the different mass numbers. For example, there are three
proof. When Chadwick bombarded a thin sheet of isotopes of hydrogen. One, simply known as
beryllium with  particles, a very high-energetic hydrogen, has one proton and no neutrons. The
radiation similar to  rays was emitted by the metal. deuterium isotope contains one proton and one
Later experiments showed that the rays actually neutron, and tritium has one proton and two
consisted of a third type of subatomic particles, neutrons. Thus, for the isotope of hydrogen, we
which Chadwick named neutrons, because they write 1H1 (hydrogen), 1H2 (deuterium) and 1H3
proved to be electrically neutral particles having a (tritium). As another example, consider two
mass (1.67493 x 10-24), slightly greater than that of common isotopes of uranium with mass numbers of
protons. 235 and 238, respectively: 92U235, 92U238.
2.3 Atomic number, Mass number and Isotopes N
2.4 Nature of light and Electromagnetic radiation
Atomic number
2.4.1 Corpuscular Theory
The number of protons and neutrons they contain
Earliest view of light due to Newton regarded light
can identify all atoms. The atomic number (Z) is the
as made up of particles (corpuscles). This is due to
number of protons in the nucleus of each atom of an
element. In a neutral atom the number of protons is

Atomic Structure & Basics of Chemistry Page 14


some facts such as reflection and refraction. But it is forms of electromagnetic radiation. The point of
failed to explain interference and diffraction. maximum upward displacement is called crest, and
Hygens proposed wave like character of light maximum downward displacement is called trough.
explains the phenomenon of interference and Thus, waves may be considered as a continuous
diffraction. In 1856 James Clark Maxwell proposed sequence of alternating crest and troughs.
that light in the form of waves. These waves have 2.4.4 Characteristics of wave motion
electric and magnetic field associated with them, A wave was characterized by its wavelength and
therefore electromagnetic radiations or frequency.
electromagnetic waves. 1. Wave length ()
2.4.2 Characteristic of electromagnetic radiation The wavelength denoted by  (lambda) is defined as
1. These consist of electric and magnetic fields that the distance between any two adjacent peaks (crests)
oscillate in the directions perpendicular to each or troughs on successive waves.
other. Unit – A0 (Angstron) 1A0 = 10-8 cm = 10-10 m
2. All electromagnetic waves travel with same SI unit:  (micrometer), m (milli micron) ,
velocity. The velocity equal to that of light, nm (nanometer), pm (picometer).
3x108 m/sec. 1A0 = 10-10m; 1 = 10-6m; 1m = 10-9m
3. These electromagnetic radiations donot require 1nm = 10-9m; 1pm=10-12m
any medium for propagation. E.g. light reaches
2. Frequency ()
us from the Sun through an empty space
The frequency,  (nu) is the number of waves or
2.4.3 Properties of Waves and Electromagnetic
cycles per second pass through a given point.
radiation
According to Rutherford’s model, an atom consists In following figure shows the frequency corresponds
of a nucleus is many times smaller than the atom to the number of times per second that moves
itself, with electrons occupying the remaining space. through a complete cycle of upward and downward
Each element has a characteristic line spectrum motion. The frequency is expressed in the unit of
because of the emission of light from atom in the hot cycles per second (s-1) or Hertz (Hz).
gas. The spectra can be used to identify elements. 1 Hz = 1 cycles per second  = C/
This spectrum of each element was characterized 3. Amplitude: It is the vertical distance from the
through the waves (i.e. electromagnetic radiation). midline of a wave to the peak or trough. Height of
Wave motion crest or depth of trough i.e. intensity or brightness.
A familiar example of a electromagnetic waves like 4. Velocity (C): Distance travelled by a light in one
waves in the surface of water. The waves originate second. The unit of velocity is ms-1 or cms-1
from the centre of disturbance and propagate in the 5. Wave number (): The number of waves per unit
form of up and down movements. length. Unit: cm-1 or m-1,  = 1/
That is, if we drop a stone into one end of quiet
pond, the impact of the stone with water starts an Relationship: C =  x  ;  = C/
up-and-down motion of the water surface. This up- 1/  =  ;  = C 
and-down motion travels outward from where the
stone hit. Exercise.1 Calculate the speed of a wave whose
A wave is a continuously repeating change or wavelength and frequency are 17.4cm and 87.4 Hz,
oscillation in matter at regular intervals. Light is also respectively.
a wave; it consists of oscillations in electric and This is directly solved by the application of c= 
magnetic fields that can travel through space. c= 17.4 cm x 87.4 Hz ; c = 17.4cm x 87.4/s
Visible light, X rays and radio waves all are the = 1.52 x 103 cm/s

Atomic Structure & Basics of Chemistry Page 15


2.5 Electromagnetic Radiation planes, and his model describes how energy in the
There are many kinds of waves, such as form of radiation can be propagated through space
water waves, sound waves, and light waves. In as vibrating electric and magnetic fields.
1873 James Clerk Maxwell proposed that light Electromagnetic radiation is the emission and
consists of electromagnetic waves. According to transmission of energy in the form of
Maxwell’s theory, an electromagnetic wave has an electromagnetic waves.
electric field component and a magnetic field
component. These two components have the same
wavelength and frequency, and hence the same
speed, but they travel in mutually perpendicular

10-12 10-11 10-10 10-8 10-7 10-6 10-4 10-3 10-1


Wavelength
(nm)  rays X rays Far Near Vis Near Far IR Microwave Radio
UV UV IR Radar TV, FM
Frequency (Hz) 1020 1018 1016 1015 1014 1012 1010

Fig.1a Types of electromagnetic radiation

Violet Red
400 nm 500nm 600nm 700nm

Fig.1b Visible light ranges from 400nm (violet) to 700nm (red)

The fig.1a shows various types of electromagnetic than 800nm and ultraviolet radiation has wavelengths
radiation, which differ from one another in less than 400nm.
wavelength and frequency. The range of 2.6 Quantum effects and photons
frequencies or wavelengths of electromagnetic Issac Newton, who studied the properties of light in
radiation is called the electromagnetic spectrum. the 17th century, believed that light consisted of a
The motions of electrons within atoms and beam of particles. In 1801, British physicist Thomas
molecules produce the shorter, visible light waves. Young showed that light like waves could be
The shortest waves, which also have the highest diffracted.
frequency, are associated with  rays, which result By the early part of the twentieth century, the wave
from changes within the nucleus of the atom. theory of light appeared to be well entrenched. But in
In fig.1b visible light extends from the violet end 1905, the German physicist Albert Einstein
of the spectrum, which has a wavelength of about discovered that he could explain a phenomenon
400nm, to the red end, with a wavelength of less known as the photoelectric effect by postulating that
than 800 nm. Beyond these extremes, light had both wave and particle properties. Einstein
electromagnetic radiation is not visible to the based this idea on the work of the German physicist
human eye. IR radiation has wavelengths greater Max Planck.

Atomic Structure & Basics of Chemistry Page 16


2.6.1 Planck’s Quantization Energy 2.6.2 Photoelectric effect
Max Planck found a theoretical formula that Einstein used this photon concept to explain the
exactly describes the intensity of light of various photoelectric effect.
frequencies emitted by a hot solid at different The photoelectric effect is the ejection of electrons
temperatures. from the surface of a metal or from another material
According to Planck, the atoms of the solid when light shines on it. Electrons are ejected, only
oscillate, or vibrate with a definite frequency,, when the frequency of light exceeds at a certain
depending on the solid and he found it necessary threshold value of the particular metal.
strong idea. For example, violet light will cause potassium metal
1. Radiant energy is not emitted or absorbed to eject electrons; not in red light (which has lower
continuously but discontinuously in the form of frequency).
small pockets of energy called quanta. Each When the photon hits the metal, its energy h is takes
such quanta is associated with a definite up by the electron. The photon cease to exist as a
amount of energy. particle; it is said to be absorbed and emits the
In the case of light, the quanta of energy are electron as radiations (light).
often called photon. The wave and particle pictures of light should be
2. The amount of energy associated with a regarded as complementary views, called wave-
quantum of radiation is proportional to the particle duality of light. The equation E = h 
frequency of light. displays this duality.
E  or E = h  and i.e., E is the energy of a light particles or photon and
Where, h is a Planck’s constant with the value  is the frequency of the associated wave.
6.63 x 10-34 J.s. 2.7 Spectrum
This relation found to be valid for all type of When white light from Sun is passed through a prism,
electromagnetic radiation. it split into a series of color band known as rainbow
3. The total amount of energy emitted or absorbed colors (VIBGYOR). This means that Sunlight is
by a body (matter) will be some whole number composed of collection of electromagnetic waves
multiple of quantum. having different wave length. The splitting of light
E= nh ----------- (1) n=1,2,3,………… into seven colors is known as dispersion and the
This means that a body can emit or absorb series of color bands is called a spectrum. In this
energy equal to h, 2h, 3h, ………. Or any spectrum, there is continuity of colors such a
other integral multiple of h. But cannot emit or spectrum is known as continuous spectrum.
absorb fractional value of h. 2.7.1 Atomic Spectra
We know that, C =  ;  = C/ Unlike the spectrum obtained by analyzing the
E= h C/ ---------- (2) sunlight, the spectra of atoms of elements can be
Equations (1) and (2) give the relation between made to emit energy by subjecting them to electric
energy of radiation and its frequency or discharge or by heating are not continuous
wavelength. (discontinuous).
The spectrum of atoms consists of sharp well-defined
A radiation which has higher the frequency or lines or bands corresponding to definite frequencies.
lowers the wavelength has more energy. For There are two types of atomic spectra. (i) Emission
example, violet light is larger frequency has more spectra (ii) Absorption spectra.
energy than red light which is having lower 2.7.2 Emission Spectra
frequency. Emission spectra are obtained when the radiations
emitted from substances are analyzed with the help of
spectroscope.
Atomic Structure & Basics of Chemistry Page 17
When the gases or vapor of chemical substances In 1885, J.J.Balmer developed a relationship among
are analyzed with heated by electric spark, light is the different wavelengths,  in the visible region in
emitted. The color of light depends upon the hydrogen spectrum, could be reproduced by simple
substance under investigation. Example, sodium formula:
gives a yellow light and potassium produces violet i.e., 1/ = (cm-1) = 109678 (1/22 – 1/ni2)
color. or (cm-1) = 1.097 x 107(1/22 – 1/ni2)
This type of spectrum consists of sharp well- ni is an integer but greater than 2, ni = 3,4,5,…..
defined line each corresponding to a definite These series of lines which appear in visible region
frequency (or wavelength). Such spectrum is are named as Balmer series.
called line spectrum of discontinuous spectrum. Afterwards, a series of spectral lines of H2 atom in
The line spectrum also known as atomic spectrum different regions were discovered. These series of
because, it is obtained from atoms by the lines named after the name of its discoverers. These
application of heat or other energy (electric). are Lyman, Balmer, Paschen, Brackett and Pfund
Each element gives a unique spectrum irrespective series.
of even the form in which it present. Example, Lyman – UV region ; Balmer – Visible
Sodium always gives lines at 589nm and 589.6nm Paschen, Brackett and Pfund – IR region
(yellow). From this reason line spectra are also As the other series of hydrogen spectral lines were
called as finger prints of atoms. discovered, a more general expression was found by
2.7.3 Absorption Spectra Rydberg, and is known as Rydberg equation derived
When a continuous electromagnetic radiation as,
(white light) is allowed to pass through a gas or a E = - RH / n2 ------------ (1) n = 1,2,3,.., 
solution of some salt and the transmitted light is (for hydrogen atom)
analyzed. We obtain a spectrum in which dark Where, RH is Rydberg constant has the value
lines are observed. These dark lines indicate that 2.18x10-18J
the radiations of corresponding wavelength have n is principal quantum number
been absorbed by the substances from the white negative sign indicates arbitrary convention.
light. The energy levels of the electron in the hydrogen
Such a spectrum containing few dark lines due to atom do undergo a transition of electron between
absorption of light is known as absorption energy levels; electron loses energy, which is emitted
spectrum. The dark lines of wavelengths are also as photon. The difference between the energies of the
character of a substance. Example, absorption of initial and final state is:
Na consists of lines at 589nm and 589.6nm same
E = Ef – Ei or Ei - Ef
as that of emission spectrum.
From equation (1), Ef = - RH(1/nf2) and
Ei = - RH (1/ni2)
2.7.4 Emission Spectrum of hydrogen atom
Therefore, E = - RH / nf2 – (- RH/ni2)
The spectrum of hydrogen atom has played a very
= RH/ni2 – RH/nf2
important role in the development of atomic
E = RH [1/ni2 – 1/nf2] --------- (2)
structure. The spectrum of hydrogen atom can be
Because this transition results in the emission of a
obtained by passing an electric discharge current
through the hydrogen gas taken in the discharge photon of frequency,  and energy h, we can write:
tube under low pressure. E = h = RH [1/ni2 – 1/nf2]
The emitted light (radiation) is analyzed by and, we have  = c/, we can write this
spectroscope. The spectrum consists of large hC/ = RH [1/ni2 – 1/nf2]
number of lines appearing in different wavelengths 1/ =  = RH/hC [1/ni2 – 1/nf2] ------- (3)
(visible, UV, IR regions).

Atomic Structure & Basics of Chemistry Page 18


When a photon is emitted, ni > nf. Consequently amount of energy. Hence, stationary states also
the term is negative and E is positive. called as energy levels.
If when energy is absorbed, ni < nf and the term is The energy associated with different energy levels
increases with increase in distance from the nucleus.
positive, so E is positive.
The letters of orbit (shell) : K,L,M,N,……
The brightness of a spectral line depends on how
The numbers of orbit (shell) : 1,2,3,4,………
many photons of the same wavelength are emitted.
are used to designate the energy levels.
In equation (3) by substituting RH=2.18x10-18J, 4. Energy associated with an energy level is given by
h=6.623x10-34 J.sec. and C=3x108m/sec, we found the relation:
that RH/hC is 1.097x107 m, which is the constant - 2π2k2Z2me4
given in the Balmer equation. En = -------------------------
The emission spectrum of hydrogen includes a n2h2
wide range of wavelengths from the IR to UV, and Where, Z – atomic number
lists the series of transitions in the hydrogen m – mass of electron
spectrum; they are named as, h – Planck’s constant
e –charge of electron
n – orbit number, and k = 9 x 109 Nm2c-2
Series nf ni Spectral region
k = 1/ 4 π0 ;
Lyman 1 2,3,4,…… Ultraviolet
0 being permittivity of empty space
Balmer 2 3,4,5,…… Visible and UV
Paschen 3 4,5,6,……. IR
For hydrogen atom, Z = 1;
Brackett 4 5,6,7,……. IR
- 2π2k2me4
Pfund 5 6,7,8,……. IR En = -------------------------
n2h2
The above equation (3), and lines only applicable substituting the value of m, e, π, k and h:
for hydrogen atom. But other atoms lines are EH = - 1312 / n2 kJ/mol
complicated and equation is also not obeyed. The For He+, Z = 2  EHe+ = 4 EH
energy levels of hydrogen atom with series of
emission spectral transition are labeled with their For Li2+, Z = 3  ELi+ = 9 EH
principal quantum numbers. 1. Different energy levels are not equally spaced. i.e.,
The spectrum of hydrogen atom holds the key to the energy difference between two successive
the inner structure of atom. Bohr proposed the energy levels is not same. It goes on decreasing
theory of atom based on Planck’s quantum theory with increasing the value of n.
and spectra of hydrogen atom. 2. Angular momentum of an electron is a whole
2.8 Bohr model of the atom number multiple of h/2π.
1. Maxwell’s law is not applicable to electron in i.e. mvr = nh / 2π
an atom. h - Planck’s constant; m- mass of electron
2. An atom consists of a massive positively v - tangential velocity;
charged nucleus. The electrons revolving r - radius of the orbit and, r = nh/2πmv
around the nucleus in a certain circular orbits The velocity of electron in nth orbit of an atom is
without radiation energy. These non-radiating given by vn = 2πe2Z / nh, and
orbits are known as stationary states. r = n2h2 / 4π2mZe2
3. Each orbit is at different distance from the substituting the value of h, π, m, Z and e for
nucleus and they are associated with definite hydrogen, Z=1 comes to be 0.53A0

Atomic Structure & Basics of Chemistry Page 19


3. Because of electron transition between two wavelength of the wave must fit exactly with
stationary states of E1 and E2, then the energy is circumferences of the orbit.
emitted or absorbed in the form of quanta. The relation between the circumference of an allowed
The frequency,  of the emitted or absorbed orbit (2r) and the wavelength () of the electron is
radiation is given by, h = E2 – E1 given by,
4. Since energy cannot be lost continuously, and 2r = n ---------------- (1)
an electron continuously to move in a particular Where, r is the radius of the orbit,  is the wavelength
energy level without losing energy. Such a state of the electron wave and n=1,2,3, ….. So, energy of
of the atom is known as ground state. electron depends on size of the orbit, its value must
On gaining energy from an external source be quantized.
electron jumps from a lower energy level to a Thus, the waves can behave like particles and
higher energy level is known as excited state. particles can exhibit wavelike properties. So, De
However the excited state is unstable, and Broglie deduced that the particle and wave properties
excited electron jumps down from higher to are related by the expression,
lower energy level (either directly or in steps)  = h/mu ----------------- (2)
by losing energy in the form of electromagnetic Where, m and u are the wave length associated with
radiation. This accounts for the spectral lines in a moving particle. Note that the left side of equation
the hydrogen spectrum. (2) involves the wavelike property of wavelength,
2.8.1 Limitations of Bohr’s Theory: whereas the right side makes references to mass, a
1. It is applicable only to hydrogen atom or distinct property of particle.
hydrogen like ions which contain only one Workout:
electron. 1. Calculate the de-Broglie wavelength of a ball of
2. The spectra of multi electron system cannot be mass 0.1kg moving with a velocity 100ms-
explained by Bohr’s theory. (Ans: 6.626x10-35m).
3. The experimental value of ionization energy 2. The mass of an electron is 9.1x10-31kg. The
and the value calculated from Bohr’s theory velocity is 800 ms-1. Calculate its wavelength.
does not agree. 2.10 Quantum mechanics
4. Bohr Theory cannot explain the mode of Bohr’s approach did not account for the emission
formation of bonds between atoms. spectra of atoms containing more than one electron,
5. It gives the flat model of the orbit. such as atoms of He and Li. He did not explain why
6. By using spectroscope of high resolving power,
extra lines appear in the hydrogen emission spectrum
it is observed that each line in the ordinary
when a magnetic field is applied and could not define
spectrum is split into a number of component the precise location of a wave.
lines, differing slight in their frequencies, this is Describe the problem of trying to locate a subatomic
called fine structure of spectral lines. Bohr’s particle that behaves like a wave, Heisenberg
theory fails to explain the fine spectrum. formulated the uncertainty principle.
7. Bohr’s theory predicts definite orbits for 2.10.1 Heisenberg Uncertainty Principle
electrons. This goes against the modern ideas of It stated that it is impossible to know simultaneously
wave nature of electron. both the momentum (p) and the position (x) of a
2.9 Dual Nature of the Electron particle with certainty. It is stated mathematically,
De Broglie stated that if light waves can behave x p  h/4 ---------------------- (3)
like a stream of particles (photons), then particles Where x and p are the uncertainties in measuring
such as electrons could possess wave properties. the position and momentum.
He argued that if an electron does behave like a
standing wave in the hydrogen atom, the
Atomic Structure & Basics of Chemistry Page 20
Applying Heisenberg uncertainty principle to the (E=H) began a new area in physics and chemistry
H atom, we see that in reality the electron does not called the quantum mechanics (wave mechanics).
in orbit the nucleus in a well-defined path, as Bohr 2.11 The Quantum mechanical Description of the
thought. hydrogen atom
If it did, we could determine precisely the position The Schrodinger equation specifies the possible
of the electron and its momentum at the same energy states of the electron can occupy in a
time, a violation of the uncertainty principle. hydrogen atom and identifies the corresponding wave
Significance: functions (). These energy states and wave
1. The principle is no significance for macro functions are characterized by a set of quantum
objects. Because x p for ion ball having numbers, with which we can construct a model of the
mass 100. Example, x v= h/4m hydrogen atom.
= 6.623x10-27 erg.sec / 4x3.142x100 In quantum mechanics, concept of electron density
= 0.526x10-29 cm2sec-1 gives the probability that an electron will be found in
2. The principle cannot be neglected for a particular region describes the atom.
microscopic particle like electron. Example, The square of the wave functions, 2, defines the
x v for electron having mass(m) is distribution of electron density in three-dimensional
9.1x10-28g space around the nucleus.
x v = h/4m = 6.62x10-27 erg.sec According to quantum mechanics, an atomic orbital
------------------------------- can be thought as the wave function of an electron in
4x3.142x9.1x10-28 kg an atom. When we say that an electron is in a certain
It is applicable and cannot be ignored. orbital, it means that the distribution of the electron
Uncertainty principle and idea of probability density or the probability of finding the electron in
picture of an atom. space is described by the square of the wave function
According to bohr’s model, electrons revolved in (2) associated with an orbital. Therefore, an atomic
well defined circular orbits. But uncertainty orbital has a characteristic energy, as well as a
principle is not possible to know the exact position characteristic distribution of electron density. “An
and path of electron. This new approach called orbital is defined as a region in space around the
wave mechanics. nucleus where the probability of finding the electron
2.10.2 Schrodinger wave mechanics is maximum”.
In 1927 Erwin Schrodinger (Austrian physicist) The Schrodinger equation is applicable nicely for the
used a complicated mathematical technique simple hydrogen atom with its one proton and one
formulated an equation that describes the behavior electron, but it turns out that it cannot be solved
and energies of submicroscopic particles. exactly for many-electron atoms. So, chemists and
In general the Schrodinger equation requires physicist have learned to get the solution for this kind
advanced calculus to solve and it is important to by applying approximation methods. So, they
know that the equation incorporates both particles concluded that the behavior of electrons in many-
behavior in terms of mass, m and wave behavior in electron atoms is not the same as in the hydrogen
terms of wave function  (psi), which depends on atom.
the location in space of the system. 2.12 Quantum Numbers
According to wave theory, the intensity of light is In quantum mechanics, three quantum numbers are
proportional to the square of the amplitude of the required to describe the distribution of electrons in
wave or 2. The most likely to find a photon is atoms. These numbers are derived from the
where the intensity is greatest, that is, where the mathematical solution of the Schrodinger equation
value of 2 is greatest. Schrodinger wave equation for the hydrogen atom. They are called the principal

Atomic Structure & Basics of Chemistry Page 21


quantum number (n), the angular momentum Letter s p d f g h
quantum number (l) and the magnetic quantum l 0 1 2 3 4 5
number (ml). n = 2, l = 0,1 has two sub shells (i.e. s, p)
These quantum numbers will be used to describe n = 3, l = 0,1,2 has 3 sub shells (i.e. s, p, d)
atomic orbitals and label the electrons. A fourth n = 4, l = 0,1,2,3 has four sub shells (i.e. s, p, d, f)
quantum number, spin quantum number describes Thus, if l=0, we have an s orbital; if l=1, we have a s
the behavior of specific electron. and p orbital and so on.
2.12.1 Principal quantum number (n) We denote a sub-shell with a particular shell, we
The principal quantum number (n) can have write the value of n quantum number for the shell,
integer values 1,2,3,……… The value of n followed by the letter designation for the sub-shell.
determines the energy of an orbital. In the case of Thus, n=1, l=0 has one sub shell ---- s
hydrogen atom or single electron atomic ions, such and, maximum number of electrons in each sub-shell:
as Li2+ and Be3+, the energy of these determined s - 2; p - 6 ; d-10 and f - 14.
by their principal quantum number. For other For example, 2p denotes a sub-shell with quantum
atoms, the energy also depends on the angular numbers n = 2 and l = 1.
momentum quantum number (l). Correlation between n and l:
The size of an atom also depends on the value of --------------------------------------------------------------
n. The orbitals of the same quantum state “n” are n l sub-shell no.of sub-shell no.of eles.
said to belong to the same shell. Shells are --------------------------------------------------------------
designates by the following letters, 1 0 1s one 2
Letter K L M N O P 2 0, 1 2s, 2p two 8
n 1 2 3 4 5 6 3 0, 1, 2 3s, 3p, 3d three 18
4 0, 1, 2, 3 4s, 4p, 4d, 4f four 32
2.12.2 Angular momentum or Azimuthal quantum --------------------------------------------------------------
number 2.12.3 Magnetic quantum number
This quantum number distinguishes the orbitals of The magnetic quantum number (ml) describes the
a given “n” having different shapes. The values orientation of the orbital in space. Within a sub-shell,
of l depends on the value of principal quantum the value of ml depends on the value of the angular
number, it can have any integer value from 0 to n- momentum quantum number, l. The allowed values
1. With each shell of quantum number, n, there are are the integers as follows:
n different kinds of orbitals, each with a different -l,….. ,0,……+l (or) –l,(-l+1),…,0,---. (+l-1),+l
shape denoted by an “l” quantum number. For For l=0 (s subshell), the allowed ml quantum number
example, is 0 only; ther is only one orbital in the s sub-shell.
If, n = 1 has only one l i.e., l=0 For l=1 (p subshell), ml=-1,0 and +1; there are 3
n = 2 has two l values i.e., l=0,1 different orbitals in the p sub-shell. The orbitals have
n = 3 has three l values i.e., l=0,1,2 the same sub-shell, but it has different orientation in
n = 4 has four l values i.e., l=0,1,2,3 space (as in px, py and pz).
An electron has a n value is 3, the possible value if, n=2, ml = -2,-1,0,+1,+2 has five orientation: dxy,
for l are 0,1 and 2. Thus, with in the M shell (n=3), dyz, dxz, dx2 - y2, dz2.
there are three kinds of orbitals, each having if, n=3, ml=-3,-2,-1,0,+1,+2,+3 has seven orbital
different shape. orientations. But the orientation of ‘f’ orbital is
The energy also depends on the “l” quantum difficult to suggest.
number, orbitals of the same ‘n’ but different, ‘l’ In addition each orbital of a given sub-shell has the
values are said to belong different sub-shells of a same energy, note there are (2l+1) orbitals in each
given shell, usually denoted by letters as, sub-shell of quantum number.

Atomic Structure & Basics of Chemistry Page 22


2.12.4 Spin quantum number
According to electromagnetic theory, a spinning
charge generates a magnetic field, and it is this
motion that causes an electron to behave like a
magnet.
The following figure shows the two possible
spinning motions of an electron, one clockwise
and the other counter clockwise. It is necessary to
introduce a fourth quantum number, called the
electron spin quantum number (ms), which has a Figure.1 Orbital energy levels in H2 atom
value of +1/2 or –1/2.
2.13 Atomic Orbitals The hydrogen’s electron has the same energy whether
The following table shows the relation between it is in the 2s orbital or a 2p orbital. The 1s orbital in a
permissible quantum numbers and atomic hydrogen atom corresponds to the most stable
orbital’s. We see that when l=0, (2l+1) = 1 and condition, the ground state. An electron residing in
there is only one value of ml, thus we have an s this orbital is most strongly held by the nucleus
orbital. When l=1, (2l+1) = 3, so there are three because it is closest to the nucleus. An electron in the
values of ml or three p orbital’s, labeled px, py and 2s,2p or higher orbital’s in a hydrogen atom is in an
pz. When l=2, (2l+1)=5 and there are five values of excited state.
ml, and the corresponding five d orbital’s are
labeled with more elaborate subscripts.
Table: Relation between quantum numbers and
atomic orbitals
-------------------------------------------------------------
n l m Number of orbital
Atomic orbital Designations
-------------------------------------------------------------
1 0 0 1 1s
Figure.2 Orbital energy levels for many e- atom
2 0 0 1 2s
The energy picture is more complex for many-
1 -1, 0, 1 3 2px, 2py, 2pz
electron atoms than for hydrogen. The energy of an
3 0 0 1 3s
electron in such an atom depends on its angular
1 -1, 0, 1 3 3px, 3py, 3pz
momentum quantum number as well as on its
2 -2,-1, 0, 1, 2 5 3dxy, 3dyz, 3dxz
principal quantum number (shown in figure.2). For
3dx2- y2, 3dz2
many electron atoms, the 3d energy level is very
--------------------------------------------------------------
close to the 4s energy level. Due to the repulsion
2.13.1 The energies of Orbitals
between the electrons, it turns out that the total
We know that, En=-RH (1/n2), from this
energy of an atom is lower when the 4s subshell is
equation the energy of an electron in a hydrogen
filled before a 3d subshell. Figure.3 depicts the order
atom is determined by its principal quantum
in which atomic orbital’s are filled in a many-electron
number. Thus the energies of hydrogen orbital
atom.
increase as follows,
1s  2s = 2p 3s=3p=3d  4s=4p=4d=4f  ……..

Atomic Structure & Basics of Chemistry Page 23


of l) of angular momentum quantum numbers 0 and
1.
 Each subshell of quantum number l contains 2l+1
orbital’s. For example, if l=1, then there are three
p orbital’s.
 No more than two electrons can be placed in each
orbital. Therefore, the maximum number of
electrons is simply twice the number of orbitals
that are employed.
 A quick way to determine the maximum number
Figure.3 The order in which atomic sub shells of electrons that an atom can have in a principal
are filled in a many electron atom level ‘n’ is to use the formula 2n2.
The increasing order of the orbital as follows, 2s  2p  3s  3p  4s  3d  4p  5s  4d  5p  6s 
1s  2s  2p  3s  3p  4s  3d  4p  5s  4d  5d  6p  7s  5f  6d  7p  6f  7d  7f  ………
5p  6s  4f  5d  6p  7s  5f  6d  7p  6f 
The following example illustrates the procedure to
7d  7f  ………
calculating the number of electrons in orbital and
2.13.2 Electron Configuration
labeling electrons with the four quantum numbers.
By using the four quantum numbers n, l,ml and ms,
i.e. What is the maximum number of electrons that
label completely an electron in any atom. For
can be present in the principal level for which n=3?
example the four quantum numbers for a 2s orbital
When n=3, then l=0, 1 and 2. The number of orbitals
electron are n = 2, l = 0, ml = 0 and ms= +1/2 and
for each value of l is given by
ms = -1/2. It is convenient to write out all the
Value of l Number of orbital’s (2l+1)
individual quantum numbers, and so we use the
0 1
simplified notation (n, l, ml, ms).
For example, above, their quantum numbers are 1 3
either (2, 0, 0, +1/2) or (2, 0, 0, -1/2). The value of 2 5
ms has no effect on the energy, size, shape or The total number of orbital’s is nine. Since each
orientation of an orbital. orbital can accommodate two electrons, the
In the case of hydrogen atom, contains only one maximum number of electrons that can reside in the
electron. The electron may be in the 1s orbital orbital’s is 2x9 or 18 or using the formula 2n2, we
(ground state) or it may be found in some higher- have n=3 and so 2(3)2 = 18.
energy orbital (an excited state). For many 2.13.4 Method of writing electronic configuration:
electron atoms, we must know the electron nlx method and Box method
configuration of the atom is how the electrons Electronic configuration is expressed by indicating
distributed among the various atomic orbitals. the principal quantum number and its representative
2.13.3 General Rules for Assigning Electrons to sub-shell along with number of electrons present in it.
Atomic Orbitals For example, in ground state hydrogen atom must be
We can formulate some general rules for in the 1s orbital, so its electron configuration is 1s1.
determining the maximum number of electrons
that can be assigned the various subshells and
orbital’s for a given value of n.
 Each shell or principal level of quantum
number n contains n subshells. For example, if
n=2, then there are two subshells (two values

Atomic Structure & Basics of Chemistry Page 24


A box for an orbital (square or circular) an arrow 1s2 2s2 2px2 2py2 2pz1
for an electron, the direction of arrow giving F atom
orientation of spin. Example, electron 1s2 2s2 2px2 2py2 2pz2
configuration of carbon is: Ne atom
1s2 2s2 2px1 2py1 2pz

Based on the above rules the electronic configuration


In the ground state electronic configuration of an of atoms having atomic number up to 30:
atom, the electrons are filled or distributed in Name Symbol Atomic Ele.configuration
various orbitals in order of their increasing
Number
energies. Thus, an orbital with lowest energy will
Hydrogen H 1 1s1
be filled first.
The electrons occupy the various orbital’s can be Helium He 2 1s2
easily remembered with the help of Moeller’s Lithium Li 3 1s22s1
diagram, and increasing order of atomic orbitals: Beryllium Be 4 1s22s2
2.14 Pauli Exclusion Principle
Boron B 5 1s22s22p1
In the case of many electron systems we can use
Carbon C 6 1s22s22p2
the Pauli Exclusion Principle to determine electron
configurations. It states that no electrons in an Nitrogen N 7 1s22s22p3
atom can have the same set of four quantum and soon……..
numbers. If two electrons may occupy in the same 2.15.1 Paramagnetism and Diamagnatism
atomic orbital, and these electrons must have If the two electrons in the 1s orbital of a helium atom
opposite spins. For example we consider the had the same, or parallel spins their net magnetic
helium atom, which has two electrons are in fields would reinforce each other. Such an
opposite spin written as, arrangement would make the helium atom is
He paramagnetic one. Paramagnetic substances are those
2.15 Hund’s Rule that are attracted by a magnet.
The Hund’s rule states that the most stable On the other hand, if the electron spins are paired or
arrangement of electrons in subshell is the one antiparallel to each other, the magnetic effects cancel
with the greatest number of parallel spins. For out and the atom is diamagnetic. Diamagnetic
example, the electron configuration of carbon substances are slightly repelled by a magnet. Also by
(Z=6) is 1s2 2s2 2p2. The orbital diagram experimentally we confirmed that the helium atom is
representation of this is: diamagnetic in the ground state.
1s2 2s2 2px1 2py1 2pz Thus, an odd number of electrons must be
paramagnetic and atoms containing an even number
C atom
of electrons may be either diamagnetic or
paramagnetic. If, consider the lithium atom (Z=3)
The fact that carbon atoms are paramagnetic, each
which has three electrons, third electron has the
containing two unpaired electrons, is in accord
parallel spin (unpaired electron) compare with first
with Hund’s rule. Similarly the electronic
electron of this atom and is therefore paramagnetic.
configuration of nitrogen, oxygen, fluorine and
2.15.2 The shielding effect in Many-Electron Atoms
neon are:
Experimentally we find that the 2s orbital lies at a
1s2 2s2 2px1 2py1 2pz1
lower energy level than the 2p orbital in many-
N atom
electron atom. Why? The 2s and 2p orbitals are larger
1s2 2s2 2px2 2py1 2pz1
than the 1s orbital; an electron in either of these
O atom

Atomic Structure & Basics of Chemistry Page 25


orbital’s will spend more time away from the 2.16 The Building-Up Principle
nucleus than an electron in the 1s orbital. Thus, we This process is based on the Aufbau principle (The
can speak a 2s or 2p electron being partly shielded German word “Aufbau” means “building up”).
from the attractive force of the nucleus by the 1s The Aufbau principle states that as protons are added
electrons. The important consequences of the one to the nucleus to build up the elements, electrons
shielding effect are that it reduces the electrostatic are similarly added to the atomic orbitals. Through
attraction between protons and the electron in the this process we gain a detailed knowledge of the
2s or 2p orbital. ground state electron configuration of the elements.
The manner in which the electron density varies as The electron configuration helps us to understand and
we move from the nucleus outward depends on the predict the properties of the elements; it also explains
type of orbital. The density near the nucleus is why the periodic table works so well.
greater for the 2s electron than for the 2p electron. The following table gives the ground-state electron
In other words, a 2s electron spends more time configuration of elements from H (Z=1) through Mt
near the nucleus than a 2p electron. For this (Z=109). The electron configurations of all elements
reason, the 2s orbital is said to be penetrating than except hydrogen and helium are represented by a
the 2p orbital, and it is less shielded by the 1s noble gas core, which shows in brackets the noble gas
electrons. element that most nearly precedes the element being
In fact, for the same principal quantum number n, considered, followed
the penetrating power decreases as the angular by the symbol for the highest filled subshells in the
momentum quantum number l increases as, outermost shells for the elements sodium (Z=11)
s  p  d  f  ……… through argon (Z=18) follow a pattern similar to
Since the stability of an electron is determined by those of lithium (Z=3) through neon (Z=10).
the strength of its attraction to the nucleus, it As mentioned already, the 4s subshell is filled before
follows that a 2s electron will be lower in energy the 3d subshell in a many electron atom. Thus the
than a 2p electron. Thus, less energy is required to electron configuration of potassium (Z=19) is 1s2 2s2
remove a 2p electron than a 2s electron because a 2p6 3s2 3p6 4s1. We can simplify the electron
2p electron is not held quit as strongly by the configuration of potassium include with argon, [Ar]
nucleus. 4s1. Similarly we can write the electron configuration
The hydrogen atom has only one electron and of calcium (Z=20) as [Ar] 4s2.
therefore is not in such shielding effect. On the The elements from scantium (Z=21) to copper (Z=29)
other hand, the atoms having more than one are transition metals. Transition metals either have
electron they can cause the shielding effect by incompletely filled d subshells or readily give rise to
lower orbital electrons.The beryllium atom has cations that have incompletely filled d subshells.
diamagnetic in nature; because it has four Consider the first transition metal series, from
electrons are arranged as paired configuration (1s2 scandium through copper. In this series additional
2s2) with unparallel spins. But in boron atoms has electrons are placed in the 3d orbitals, according to
paramagnetic one, due to boron has five electrons Hund’s rule. However, there are two irregularities.
are arranged as 1s2 2s2 2p1 with unpaired p The electron configuration of chromium (Z=24) is
electron is represented as: [Ar] 4s13d5 and not [Ar] 4s23d4. Similarly in copper,
whose electron configuration is [Ar] 4s13d10 and not
1s2 2s2 2px1 [Ar] 4s2 3d9. The reason for these irregularities is that
B atom a slightly greater stability is associated with the half-
filled (3d5) and completely filled (3d10) subshells.
Electrons in the same subshell have equal energy but
different spatial distributions. Consequently, their
Atomic Structure & Basics of Chemistry Page 26
shielding of one another is relatively small, and the
electrons are more strongly attracted by the nucleus
when they have the 3d5 configuration. According to
Hund’s rule, the orbital diagram for Cr is,
3d5 4s1
Cr [Ar]
Thus, Cr has a totally six unpaired electrons.
The orbital diagram for copper is,
3d10 4s1
Cu [Ar]
Again, extra stability is gained in this case by having
the 3d subshell completely filled.
For elements Zn (Z=30) through Kr (Z=36), the 4s
and 4p subshells fill in a straightforward Figure: Classification of groups of element in the
periodic table according to the type of outermost
manner. With rubidium (Z=37), electron begin to subshell being filled with electrons
enter the n=5 energy level.
Exercise.1 An oxygen atom has a total of eight
The sixth period of the periodic table begins with Cs
electrons. Write the four quantum numbers for each
(Z=55) and barium (Z=56), whose electron
of the eight electrons in the ground state.
configuration are [Xe] 6s1 and [Xe] 6s2 respectively.
1s2 2s2 2px2 2py1 2pz1
Next we come to lanthanum (Z=57), in this case
O atom
after filling the 6s orbital we would place
additional electrons in 4f orbitals. In reality, the
Solution:
energies of the 5d and 4f orbitals are very close; in
We start with n=1, so l=0, a subshell corresponding
fact, for lanthanum 4f is slightly higher in energy
to the 1s orbital. This orbital can accommodate a total
than 5d. Thus lanthanum’s electron configuration
of two electrons. Next, n=2 and l =0,1. The l=0
is [Xe] 6s2 5d1 and not [Xe] 6s24f1.Following
subshell contains one 2s orbital, which can
lanthanum are the 14 elements known as the
accomo=modate two electrons. The remaining four
lanthanides or rare earth series [ce(Z=58) to
electrons are placed in the l=1 subshell, which
lutetium (Z=71)]. The rare earth metals have
contain three 2p orbitals. The orbital diagram is,
incompletely filled 4f subshells or readily give rise
The results are summarized in the following table,
to cations that have incompletely filled 4f
----------------------------------------------------------------
subshells. Note that the electron configuration of
Electron n l ml ms Orbital
gadolinium gains extra stability by having a half-
----------------------------------------------------------------
filled subshell (4f7).
1 1 0 0 +1/2 1s
The last row of elements is the actinide series,
2 1 0 0 -1/2 1s
which starts at thorium (Z=90). Most of these
3 2 0 0 +1/2 2s
elements are not found in nature but have been
4 2 0 0 -1/2 2s
synthesized. The following figure (skeleton of
5 2 1 -1 +1/2 2px
periodic table) shows the groups the elements
6 2 1 0 +1/2 2py
according to the type of subshell in which the
7 2 1 1 +1/2 2pz
outermost electrons are placed.
8 2 1 -1 -1/2 2px
----------------------------------------------------------------

Atomic Structure & Basics of Chemistry Page 27


Chapter-III Periodic Properties elements. He realized that some of the elements did
not fit in with his scheme of classification if the order
of atomic weight was strictly followed. He ignored
The periodic table is the most important concept in the order of atomic weights, thinking that the atomic
chemistry, both in principle and practice. measurements might be incorrect, and placed the
3.1 Need to classify elements elements with similar properties together. For
At present 118 elements were known. Out of them, example, iodine with lower atomic weight than that
the recently discovered elements are man-made. of tellurium (Group VI) was placed in Group VII
With such a large number of elements it is very along with fluorine, chlorine, bromine because of
difficult to study individually the chemistry of all similarities in properties (Fig. 3.1). At the same time,
these elements and their compounds individually. keeping his primary aim of arranging the elements of
To ease out this problem, Scientists searched for a similar properties in the same group, he proposed that
systematic way to organize their knowledge by some of the elements were still undiscovered and,
classify the elements. therefore, left several gaps in the table. For example,
3.2 Brief history of modern periodic law and both gallium and germanium were unknown at the
modern periodic table time
The periodic law, as we know it today owes, it Mendeleev published his Periodic Table. He left the
development to the Russian chemist, Dmitri gap under aluminium and a gap under silicon, and
Mendeleev and the German chemist, Lother called these elements Eka-Aluminium and Eka-
Meyer. Working independently, both the chemists Silicon. Mendeleev predicted not only the existence
in 1969 proposed that on arranging elements in the of gallium and germanium, but also described some
increasing order of their atomic weights. of their general physical properties.
Similarities appear in physical and chemical 3.2.1 Modern periodic Law
properties at regular intervals. According to Henry’s Mosely, Mendeleev’s periodic
Lother meyer plotted the physical properties such law was modified. This is known as the modern
as atomic volume, melting point and boiling point periodic law and can be stated as: The physical and
against atomic weight and obtained periodically chemical properties of the elements are periodic
repeated pattern, he observed a change in length of functions of their atomic numbers.
3.2.2 Modern periodic table ( Long form of
that repeating pattern.
periodic table)
Mendeleev published the periodic law for the first
Many different forms of a periodic classification of
time, stated as; the properties of the elements are a
the elements have appeared since the 1871 periodic
periodic function of their atomic weights.
table by Mendeleev. Some forms emphasize chemical
Mendeleev arranged elements in horizontal rows
reactions and valence, whereas others stress the
and vertical columns of a table in order of their
electronic configuration of elements. A modern
increasing atomic weights in such a way that the
version, the so-called “Long form” of the periodic
elements with similar properties occupied the
table (shown in fig.1), is the most convenient and
same vertical column or group. Mendeleev’s
widely used.
system of classifying elements was more elaborate
The horizontal rows (series)of periodic table are
than that of Lothar Meyer’s. He fully recognized
called periods and the vertical columns called groups.
the significance of periodicity and used broader
Elements having similar chemical properties, and are
range of physical and chemical properties to
arranged in 1-18 horizontal rows referred to as groups
classify the elements. In particular, Mendeleev
or families.
relied on the similarities in the empirical formulas
and properties of the compounds formed by the

Atomic Structure & Basics of Chemistry Page 28


The inert gases are placed at the extreme right of is to ruthenium (Ru) and osmium (Os). Most of these
the table, reflecting the completion of the s and elements have several charges, and their ions in
p sub-shells of their outer or valence shell. solution are colored (ions of all other elements are
There are seven horizontal rows, called periods. colorless).
The period number corresponds to the highest In this region of the table, the rule of group
principal quantum number (n) of the elements in similarities loses much of its usefulness. In Group
the period. The first period contains 2 elements, IVA, for example, carbon (C) is a nonmetal; silicon
the second and third containing 8 elements, the (Si) and germanium (Ge) are semi-metals; and tin
fourth and fifth periods containing 18 elements, (Sn) and lead (Pb) are metals.
and the sixth and seventh periods containing The first element in the periodic table, hydrogen,
32 elements. This gives a total of 118 elements. does not have properties that satisfactorily place it in
In this form of the periodic table, 14 elements of any group. Hydrogen has two unique features: (a) the
both sixth and seventh periods (lanthanides and highest energy shell of a hydrogen atom can hold
actinides, respectively) are placed at the main only two electrons, in contrast to all others (except
body of the periodic table. helium) that can hold eight or more; and (b) when
Each set of elements appearing in the vertical hydrogen loses its electron, the ion formed, H+, is a
column of a periodic table is called a Group and bare nucleus. The hydrogen ion is very small in
represents a family of elements that have similar comparison with a positive ion of any other element,
physical and chemical properties. Group IA is the which must still have some electrons surrounding the
alkali family; Group IIA is the alkaline earth nucleus. Hydrogen can either gain or lose an electron.
family; Group VIA is the Oxygen family; Group It has some properties similar to Group IA elements,
VIIA is the halogen family. On the left side of the and some similar to Group VIIA elements.
table ware Group IA elements (except hydrogen), 3.3 Class of periodic elements : Based on
which are soft metals that undergo similar electronic configuration
chemical reactions. The elements in Group IIA s – Block elements
form similar compounds and are much harder than Group 1 (IA) and group 2 (IIA) which have ns1 and
their neighbors in Group IA. ns2 outermost electronic configuration belongs to the
The most accurate observations, that all elements s-block elements. They are all reactive metals with
within a particular group have similar physical and low ionization energies, readily forms +1 and +2 ion
chemical properties. This observation is most respectively. Because of high reactivity they are
accurate at the extreme sides of the table. All never found pure form in nature.
elements in Group VIIIA are unreactive gases, and p - block elements
all elements in Group VIIA have similar chemical The p-block elements comprise those belonging to
properties, although there is a gradual change in group 13-18 and these together with the s-block
physical properties. For example, fluorine (F) is a elements (except noble gases) are called
gas while iodine (I) is a solid at room temperature. representative or main group elements . The outer
Groups with a B designation (IB through VIIB) most electronic configuration varies from ns1 np1 to
and Group VIII are called transition groups. In this ns2 np6 in each period. At the end of each period
region of the table, exceptions begin to appear placed a noble gas element.
within any group in this region, all the elements Proceeding the noble gas family are two chemically
are metals, but their chemical properties may important groups of non-metals. They are the
differ. In some cases, an element may be more halogens group 17) and the chalcogens (group 16).
similar to neighbors within its period than it is to The non-metallic character increases as we move
elements in its group. For example, iron (Fe) is from left to right across a period and metallic
more similar to cobalt (Co) and nickel (Ni) than it character increases as we go down the group.
Atomic Structure & Basics of Chemistry Page 29
d – block elements 3.3.1.1 Chemistry of IA group elements (Alkali
These are the elements of group 3 - 12 in the metals)
centre of the periodic table. These elements have The alkali metals such as Lithium (Li), Sodium (Na),
the general outer electronic configuration (n-1)d1 potassium (K), Rubidium Rb), Caesium (Cs) and
to 10
ns0 to 2. They are all metals. Transition metals Francium (Fr) are members of group 1 ((IA) of the
form a bridge between the chemically active periodic table, and each has a ground state valence
metals of s-block elements and the less active electronic configuration as ns1.
elements of groups 13 - 14. Electronic configuration
f - block elements All the alkali metals have one valence electron, ns1
The two rows of elements at the bottom of the outside the noble gas core. The loosely hold s-
periodic table, called lanthanides (58Ce – 71Lu) and electron in the valence shell of these elements makes
actinides (90Th – 103Lr) are characterized by the them the more electropositive metals, M+ ions. Hence
outer electronic configuration (n-2)f1 to 14 (n- they are never found as free elemental state in nature.
0 – 1 2
1) d ns . The last electron added to each Lithium (3Li) [He] 2s1
element is filled in f – orbital. Sodium (11Na) [Ne] 3s1
These two series of elements are hence called the Potassium (19K) [Ar] 4s1
inner transition elements (f-block elements).They Rubidium (37Rb) [Kr] 5s1
are also metals. Caesium (55Cs) [Xe] 6s1
Actinide elements are radioactive. Many of the Francium (87Fr) [Rn] 7s1
actinide elements have been made only in Atomic and ionic Radii
nanogram or even less by nuclear reactions and The alkali metal atoms have the largest size than their
their chemistry is not fully explored. The elements corresponding monovalent ions (M+) i.e., Na > Na+;
after uranium are called trans-uranium elements, K > K+.
3.3.1 s - block elements By increase in atomic number, the atom of IA group
The elements in the first two groups of the elements becomes larger. Hence the atomic and ionic
periodic table are characterized as “s-block” radii of alkali metals increase on moving down the
elements because of their outer shell having one or group i.e., they increase in size while going from Li
two electrons in an s-orbital. to Cs.
The elements in group IA (alkali metal) and group Ionization energy (ionization potential)
IIA (alkaline earth metals) of the periodic table The ionization energy of the alkali metals are
have the valence shell configuration of ns1 and ns2 considerably low and decreases down the group from
respectively. These group elements are readily Li to Cs. This is because the effective nuclear charge
forms +1 and +2 ions respectively. All naturally and the outermost electron are very well screened
occurring compounds of these elements containing from the nuclear charge.
the atoms in those forms (i.e., ionic form) due to As a result of the decrease in ionization energy, the
these elements are very reactive, hence they are reactivity of the metals in group IA increase in
not found in natural as elemental state. progressing down in the group.
A large number of compounds of the group IA and The electronegativities of alkali metals range from
IIA metals occur naturally (salt, soda, limestone, 0.8 to 1.0, so all are very strong reducing agents that
etc.). These compounds have been important for form binary compounds with most non metallic
thousands of years and they are useful still. elements and even some other metals that have higher
Moreover lime is still produced enormous quantity electronegativity.
of heat upon heating the limestone. Because, the IA group metals are such a strong
reducing agents and so reactive, and they are
generally prepared by electrolysis reactions. Na is

Atomic Structure & Basics of Chemistry Page 30


produced by the electrolysis of molten mixture from the others and shows a diagonal relationship to
that contains NaCl and CaCl2. Li is produced by Al.
the electrolysis of a mixture of LiCl and KCl. The Electronic Configuration
production of K is carried out by using Na as These elements have two electrons in the s -orbital of
reducing agent at 850C under equilibrium the valence shell. Their general electronic
condition. Na(s) + K+  Na+ + K(g) configuration may be represented as [noble gas] ns2.
Negative ions Like alkali metals, the compounds of these elements
In most instances, the IA group metals are also predominantly ionic as M2+.
form +1 ions, but this is not always the case. Beryllium - 4Be [He] 2s2
Because the low ionization potential of Na, an Magnesium - 12Mg [Ne] 3s2
unusual situation exists with regard to forming Na+ Calcium - 20Ca [Ar] 4s2
and Na- ions pair as: Strontium - 38Sr [Kr] 5s2
Na(g) + Na(g)  Na+(g) + Na-(g) Barium - 56Ba [Xe] 6s2
We find that it requires 496 KJ/mol to remove an Radium - 88Ra [Rn]7s2
electron from a sodium atom (the I.P). Adding the Atomic and Ionic Radii
electron to another Na atom, Na(g) + e- Na-(g) The atomic and ionic radii of the alkaline earth metals
Hydration energy are smaller than those of the corresponding alkali
The hydration energy of alkali metal ions metals in the same periods. This is due to the
decreases with increase in ionic sizes. increased nuclear charge in these elements. Within
Li+ > Na+ > K+ > Rb+ > Cs+ the group, the atomic and ionic radii increase with
Li+ has maximum degree of hydration and for this increase in atomic number.
reason Lithium salts are mostly hydrates. Ionization Energy
e.g. LiCl . 2H2O The alkaline earth metals have low ionization
Physical properties energies due to fairly large size of the atoms. Since
 All the alkali metals are silvery white, soft the atomic size increases down the group, their
and light metals. ionization enthalpy decreases. The first
 Because of the larger size, these elements ionization energy of the alkaline earth metals are
have low density which increases down the higher than those of the corresponding Group 1
group. However K is lighter than Na. metals, e.g., Mg+ > Na+. This is due to their small
 The melting and boiling points of the alkali size as compared to the corresponding alkali metals.
metals are low indicating weak metallic It is interesting to note that the second ionization
bonding due to the presence of single energies of the alkaline earth metals (like Mg2+) are
valence electron. smaller than those of the corresponding alkali metals.
 The alkali metals and their salts impart Hydration Energy
characteristic color to an oxidizing flame. Like alkali metal ions, the hydration energies of
These elements gives a characteristic color alkaline earth metal ions decrease with increase in
under in flame as, ionic size down the group.
Li – Crimson red Rb – Red violet Be2+ > Mg2+ > Ca2+ > Sr2+ > Ba2+
Na – Yellow Cs - Blue The hydration energies of alkaline earth metal ions
K – Violet are larger than those of alkali metal ions. Thus,
3.3.1.2 Chemistry of IIA group elements compounds of alkaline earth metals are more
Elements: 4Be, 12Mg, 20Ca, 38Sr, 56Ba & 88Ra extensively hydrated than those of alkali metals, e.g.,
These elements follow alkali metals are known as MgCl2 and CaCl2 exist as MgCl2.6H2O and CaCl2·
alkaline earth metals. The first element Be differs 6H2O while NaCl and KCl does not form such
hydrates.

Atomic Structure & Basics of Chemistry Page 31


Physical Properties The colors of the oxides vary from white to orange;
 The alkaline earth metals, in general, are silvery Li2O and Na2O form white crystals while K2O is pale
white, lustrous and relatively soft but harder yellow, Rb2O yellow and Cs2O orange.
than the alkali metals. All the oxides are strong bases, the basicity
 Beryllium and magnesium appear to be increasing from Li2O to Cs2O. Peroxide of lithium
somewhat grayish. can be obtained by the action of H2O2 on an ethanolic
 The melting and boiling points of these metals solution of LiOH, but it decomposes on heating.
are higher than the corresponding alkali metals Sodium peroxide (widely used as an oxidizing agent)
due to smaller sizes. The trend is, however, not is manufactured by heating Na metal on Al trays in
systematic. Because of the low ionization air; when pure, Na2O2 is colorless and the faint
enthalpies, they are strongly electropositive in yellow color usually observed is due to the presence
nature. The electropositive character increases of small amounts of NaO2. The superoxides and
down the group from Be to Ba. peroxides contain the paramagnetic [O2]- and
 Calcium (Ca), Strontium (Sr) and Barium (Ba) diamagnetic [O2]2- ions respectively.
impart characteristic brick red, crimson and Partial oxidation of Rb and Cs at low temperatures
apple green colors to the flame. yields suboxides such as Rb6O, Rb9O2, Cs7O and
 The electrons in Be and Mg are too strongly Cs11O3. The formulae of the suboxides are misleading
bound to get excited by flame. Hence these in terms of the oxidation states. Each contains M+ and
elements do not impart any color to the flame. O2-ions, and, for example, the formula of Rb6O is
 The flame test for Ca, Sr and Ba is helpful their better written as (Rb+)6(O2-).4e- indicating the
detection in qualitative analysis ans estimation presence of free electrons.
by flame photometry. The alkali metals oxides, peroxides and superoxides
 The alkaline earth metals like those alkali react with water, strongly basic solution are produced
metals have high electrical and thermal according to equations 4-6. One use of KO2 is in
conductivities which are typical characteristic breathing masks where it absorbs H2O producing O2
of metals. for respiration and KOH, which absorbs exhaled CO2
3.4 Chemical Properties of s-block elements (reaction.7)
3.4.1 IA Group elements (Alkali metals) M2O + H2O  2MOH (4)
Oxides, peroxides, superoxides, suboxides M2O2 + 2H2O  2MOH + H2O2 (5)
and ozonides 2MO2 + 2 H2O  2MOH+ H2O2 + O2 (6)
When the group 1 metals are heated in an excess KOH + CO2  KHCO3 (7)
of air or in O2, the principal products obtained Sodium peroxide reacts with CO2 to give Na2CO3,
depend on the metal: rendering it suitable for use in air purification in
lithium oxide, Li2O (1), confined spaces (e.g. in submarines); KO2 acts
sodium peroxide, Na2O2 (2), and the similarly but more effectively. Although all the group
superoxides KO2, RbO2 and CsO2 (3). 1 peroxides decompose on heating according to
4Li + O2  2Li2O oxide formation equation 8, their thermal stabilities depend on cation
2Na + O2  Na2O2 peroxide formation size; Li2O2 is the least stable peroxide, while Cs2O2 is
K + O2  KO2 superoxide formation the most stable. The stabilities of the superoxides
The oxides Na2O, K2O, Rb2O and Cs2O can be (with respect to decomposition to M2O2 and O2)
obtained impure by using a limited air supply, but follow a similar trend.
are better prepared by thermal decomposition of M2O2(s)  M2O (s) + ½ O2(g) (8)
the peroxides or superoxides. Ozonides, MO3, containing the paramagnetic, bent
[O3]- ion, are known for all the alkali metals.

Atomic Structure & Basics of Chemistry Page 32


The salts KO3, RbO3 and CsO3 can be prepared 3.5 p-block elements
from the peroxides or superoxides by reaction with 3.5.1 Introduction
ozone, but this method fails, or gives low yields, p-block elements are placed in groups 13 to 18 of the
for LiO3 and NaO3. These ozonides have recently periodic table. Their valence shell electronic
been prepared in liquid ammonia by the configuration is ns2np1–6
interaction of CsO3 with an ion-exchange resin The properties of p-block elements are greatly
loaded with either Li+ or Na+ ions. The ozonides influenced by atomic sizes, ionization energy,
are violently explosive. electron affinity and electronegativity.
Hydroxides Groups IIIA to 0 constitute the p-block in the
When the oxygen compounds of group IA Periodic Table. The atoms of all the elements in these
and IIA metals react with water, strongly basic groups have, besides two electrons in the s-orbitals,
solutions are produced regardless of whether an one or more valence electrons in the p-orbitals.
oxide, peroxide, or superoxide is involved. Unlike the s-block elements, which show similar
Li2O + H2O  2LiOH (9) chemistry and regular variations in properties,
Na2O2 + 2H2O  2NaOH + H2O2 (10) elements of p-block, except the halogens, show more
2KO2 + 2 H2O  2KOH+ H2O2 + O2 (11) dissimilarities in their properties. In general, p-block
Sodium hydroxide, sometimes referred to as elements are mainly non-metals. Their first ionization
caustic soda or simply caustic, is produced in energies are high because their atoms have large
enormous quantities by the electrolysis of an effective nuclear charge. Therefore, they have little
aqueous solution of sodium chloride. tendency to form positive ions and are more
2 NaCl + 2 H2O + 2NaOH + Cl2 + H2 electronegative than s-block elements. When forming
The fact that this is also the reaction used to compounds with metals, p-block elements tend to
prepare chlorine makes it especially important. form negative ions by gaining electrons to attain the
However, sodium hydroxide will react with octet configuration. However, they tend to form
chlorine as shown in the equation covalent bonds by sharing electrons when combining
2OH- + Cl2  OCl- + Cl- + H2O with non-metals.
Halides 3.5.2 General aspects and trends in the p-block
elements
Sodium chloride is found in salt beds, salt
 Most p-block elements can adopt more than one
brines, and sea water throughout the world, and it
oxidation state, they normally vary in units of 2.
is also mined is some locations. Consequently,
 Group 13 compounds act as Lewis acids, as can
sodium chloride is the source of numerous other
compounds of Group 14 and 15 if they contain
sodium compounds. A large portion of the sodium
elements of the third period or lower.
chloride utilized is consumed in the production of
 Group 15, 16, and 17 compounds have lone pairs,
sodium hydroxide. The production of sodium
so they can act as Lewis bases, but the strength of
metal involves the electrolysis of the molten
the base depends on the type of orbital the electron
chloride, usually in the form of a eutectic mixture
pair is in.
with calcium chloride. Sodium carbonate is an
Trends across the periods on p-block
important material that is used in many ways such
 The atomic radius decreases, this is because of the
as making glass. It was formerly produced from
addition of electrons takes place in the same
NaCl by means of the Solvay process, in which the
valence shell and are subjected to an increased pull
overall reaction is
of the nuclear charge at each step.
NaCl(aq)+CO2(g)+H2O(l)+NH3(aq)NaHCO3(s)+NH4Cl(aq)
The valence electrons are pulled closer to the
The solid compound decomposes when heatedas
nucleus. This result in decreasing the atomic size
follows: NaHCO3(s)  Na2CO3(s) + H2O(g)

Atomic Structure & Basics of Chemistry Page 33


from as we move left to the right in periodic  The metallic character of an element decreases
elements. down each group of the Periodic Table due to
 Atomic size decreases and nuclear charge decreasing ionization enthalpies.
increases, both the factors lead to an increase  The oxidation state increases in importance down
in the force of attraction between the nucleus Groups 13–15. This is known as the inert pair
and the valence shell electrons. Thus effect.
ionization energy increases as we move from Inert pair effect: The tendency of ns2 electron
left to right in period of elements. It reaches pair to participate in bond formation decreases
maximum value at neon which has ns2np6 with the increase in atomic size. Within a group
electronic configuration (i.e stable octet) and the higher oxidation state becomes less stable with
that is why noble gases have highest ionization respect to the lower oxidation state as the atomic
energies. number increases. This trend is called ‘inert pair
 The electronegativity increases across a period. effect’. In other words, the energy required to
As the effective nuclear charge increases along unpair the electrons is more than energy released
a period the pull of the nucleus of an electron in the formation of two additional bonds.
of the other atom increases strongly.  Bond energies generally decrease down each
 The metallic character of an element decreases group, but when the atoms have lone pairs the
from left to right across a period of the p block bond enthalpy is low for the second period
due to increasing ionization enthalpies. element due to high electron–electron repulsion.
 The maximum oxidation state increases across  The ionic character of the oxides and halides
a period, though expansion of the octet is not increases down the groups and the oxides become
possible for the second period elements. more basic.
 The ionic character of the oxides and halides  pi bonding becomes less important going down
decreases across each period. each group.
 The acidic character of the oxides increases 3.6 Groups in p-block elements
across each period. 1. Boron family: Elements of group 13 – boron (B),
Trends down the groups on p block aluminium (Al), gallium (Ga), indium (In) and
 The atomic size increases, this is due to new thallium (Tl) belong to configuration is ns2np1.
electronic shell which is added when we move 2. Carbon family: Elements of group 14 – carbon
from one element to the next in group. (C), silicon (Si), germanium (Ge), tin (Sn) and
 The additional increase in the energy level lead (Pb) belong to configuration is ns2np2.
more than compensates for the added nuclear 3. Nitrogen family: The elements of group 15 –
charge. nitrogen (N), phosphorus (P), arsenic (As),
 The ionization energy decreases due to antimony (Sb) and bismuth (Bi) belong to
increasing size of the atom down the group of configuration is ns2 np3.
elements therefore, the force of attraction for 4. Oxygen family: Group 16 consists of five
the nucleus and outermost valence shell elements – oxygen (O), sulphur (S), selenium (Se),
electron decreases. Thus ionization energy is tellurium (Te) and polonium (Po). Their general
lowered. electronic configuration is ns2np4.
 Electronegativity decreases down a group. As 5. Halogen family: Group 17 elements, fluorine (F),
the atomic size increases, the bond electrons chlorine (Cl), bromine (Br), iodine (I) and astatine
are further from the nucleus and thus the force (At), belong to halogen family. Their general
of attraction between the nucleus of an atom and the electronic configuration is ns2 np5.
bond electrons decreases. 6. Group 18 elements: Helium (He), neon (Ne),
argon (Ar), krypton (Kr), xenon (Xe), and radon
Atomic Structure & Basics of Chemistry Page 34
(Rn) are Group 18 elements. They are also called amphoteric, but indium and thallium show more
noble gases. Their general electronic configuration metallic character and do not dissolve in alkalis. This
is ns2 np6. Except helium which has electronic group of elements shows a wide variation in
configuration 1s2 other elements are called noble properties. Boron is typical non-metal, aluminium is a
gases because they show very low chemical metal but shows many chemical similarities of boron,
reactivity. and gallium, Tl are due to inability of d- and f-
3.6.1 Boron Family electrons, which have low screening effect, causes
Boron is a non-metal, whereas the other Group 13 higher nuclear charge.
elements are metals. The most common oxidation 4. Electronegativity
state for Group 13 is +3, but the +1 oxidation state Down the group, electronegativity first decreases
becomes increasingly important going down the from B to Al and then increases marginally. This is
group. because of the discrepancies in atomic size of the
1. Electronic Configuration
elements.
The valence shell electronic configuration of
5. Chemical properties
these elements is ns2np1. Boron and aluminium
Due to smaller size of boron, ionization energies is
have noble gas core, gallium and indium have
noble gas plus 10 d-electrons, and thallium has very high and this prevents it to form +3 ions and
noble gas 14 f-electrons plus 10 d-electrons cores, forces it to form only covalent compounds.
Thus the electronic structures of these elements Ionization energies of Al considerably lower, and is
are more complex than for the IA and IIA group there a form able Al3+ ion. In fact, Al is a highly
elements. This difference in electronic structure electropositive metal.
affects the other properties and consequently the The relative stability of +1 oxidation state increases
chemistry of all the elements of this group. for heavier elements: Al < Ga < In < Tl.
2. Atomic radii Like BF3 molecule called electron deficient molecule
On moving down the group, for each successive sharing their electrons with others achieve stable
member one extra shell of electrons is added and,
electronic configuration and thus, behave as Lewis
therefore, atomic radius is expected to increase.
acids.
However, a deviation can be seen. Atomic radius
BCl3 easily accept lone pair of electrons from NH3, to
of Ge is less than that of Al. This can be
understand by the presence of additional 10 d- fom BCl3’NH3.
electrons offer poor screening (shielding) effect AlCl3 achieves stability by forming a dimer.
for the valence electrons from the increased 6. Diagonal relationship of boron with silicon
nuclear charge in gallium. Consequently, the 1. Both are non-metals, bad conductor of electricity,
atomic radius of gallium (135 pm) is less than that have high melting points and low densities.
of aluminium (143 pm). i.e., Al > Ga ; Ga > In; In 2. They exist in two allotropic forms viz. amorphous
> Tl. and crystalline. Their crystalline forms are harder
3. Ionization Energy than amorphous forms.
The ionization energy values as expected from the 3. They do not form such an oxy-salt in which boron
general trends do not decrease smoothly down the
and silicon may present in cationic form.
group. The decrease from B to Al is associated
4. Their oxides (B2O3 and SiO2) are acidic in nature,
with increase in size. The observed discontinuity
dissolve in alkalis to form borate and silicate.
in the ionization energy values between Al and
Ga, and between In and Aluminium and gallium B2O3 + 6 NaOH  2Na3BiO3 + 3H2O
are SiO2 + 2 NaOH  Na2SiO3 + H2O
They can reduce by Mg to B and Si.

Atomic Structure & Basics of Chemistry Page 35


B2O3 + 3 Mg  2B + 3MgO 4. Boron forms covalent compounds only, while
SiO2 + 2Mg  Si + 2MgO other family members form electrovalent
5. Both form a number of covalent hydrides which compound also.
have similar properties e.g. 5. Boron does not form hydrated ions of the type
Boron: Diborane (B2H6), Tetraborane (B4H10) [M(H2O)6]+.
etc. 6. Oxides and hydroxides of boron are acidic, those
Silicon: Silane (SiH4), Disilane (Si2H6) etc. those aluminimum and gallium are amphoteric
6. Both burn in halogens to form halides which
while those of indium and thallium are basic.
are co-valent in nature. Fluorides of both are
7. Boron forms electron deficient compounds in
hydrolysed by water forming acids.
which it acts as electron acceptors (Lewis acid)
4BF3 + 2H2O  HBO2 + 3HB4
while the other of family members do not show
Meta boric Hydrofluo
this property.
acid boric acid
3SiF4 + 3H2O  H2SiO3 + 2H2SiF6
3.6.2 Elements of group IV A (Carbon family)
Silicic acid Hydrofluo
The IVA sub group of the periodic table contains five
silicic acid
elements viz. Carbon (C), Silicon (Si), Germanium
The chlorides of both (BCl3 and SiCl4) are liquids,
(Ge), Tin (Sn) and Lead (Pb). They are p-block
fume in moist air and are hydrolyzed by water. elements and also known as Carbon family.
BCl3 + 3H2O  H3BO3 + 3HCl 3.6.2.1 Similarities and gradation in physical
SiCl4 + 4H2O  H4SiO4 + 4HCl properties
7. The carbides of both the elements (B4C and 1. Electronic configuration
SiC) are very hard substances and used as The electronic configurations of these elements are
abrasive. as follows:
2 2
6C = [He] 2s 2p
8. Both dissolve in alkalis forming borate and 2 2
14Si = [Ne] 3s 3p
silicate. 10 2 2
32Ge = [Ar] 3d 4s 4p
2B + 6NaOH  2Na3BO3 + 3H2 10 2 2
50Sn = [Kr] 4d 5s 5p
Si + 2NaOH + H2O  Na2SiO3 + 2H2 14 10 2 2
82Pb = [Xe] 4f 5d 6s 6p
9. Both form complexes e.g. hydrofluoboric acid, It appears that these elements have ns2np2
HBF4 and hydrofluosilicic acid, H2SiF6 electrons in their valence shell.
2. Metallic character: The metallic character
Anomalous Behavior of Boron increases as we move down the group e.g. carbon
Boron and its compounds differ from other family and silicon are non-metals, germanium is a
members in the following properties. metalloid while tin and lead is metals.
1. Boron is non-metallic in nature while other 3. Atomic radius: Increases gradually from carbon
to lead, even for poor ability to screening the
members are either metalloids or metals.
nucleus from the valence electrons due to metallic
2. Melting point of boron (23000C) is highest in
nature increases down the group.
comparison with the melting point of other
4. Ionization Energy: The value of ionization
family members. energy decreases gradually on moving from C to
3. It has lowest value for atomic radius and Sn. The I.E for Pb is slightly greater due to poor
highest value of ionization potential. screening (shielding) effect of f-orbitals of
metallic lead.

Atomic Structure & Basics of Chemistry Page 36


5. Conductivity:Carbon (diamond) bad conductor, silicon and germanium are semi-conductors
while tin and lead are good conductors.
6. Oxidation states: Carbon and silicon have +4
oxidation state, while the remaining elements of this group show +2 and +4 oxidation states.
The stability of +4 oxidation state decreases while that of +2 oxidation state increases.
7. Allotropy: All these elements (except Pb) show allotropy. Carbon exists in two important
crystalline forms – diamond and graphite.
Silicon exists in crystalline and amorphous
forms while germanium exists in two crystalline forms. Tin exists in three solid forms:
Grey (α) tin  white () tin  brittle tin
8. Diagonal relationship between carbon (C) and phosphorous (P)
Carbon (IVA) is diagonally related to phosphorous (VA) in the following respect:
1. Both are reactive non-metallic elements.
2. Both the elements show allotropy
3. Carbon and phosphorous form covalent
compounds.
4. Oxides of both are acidic in nature.
5. Oxy-acids and their salts of both the
elements are stable.
6. Hydrides, nitrides and carbides are covalent.
7. Both have tendency of catenation.

3.6.3 Elements of VA (15th) group (Nitrogen family)


The VA sub group of the periodic table contains five elements viz. nitrogen (N), phosphorous
(P), arsenic (As), antimony (Sb) and bismuth (Bi). They are also known as nitrogen family.
3.6.3.1 Similarities & gradation in physical properties
1. Electronic configuration (valence
electrons ns2np3)
2 3
7N = [He] 2s 2p
2 3
15P = [Ne] 3s 3p
10 2 3
33As= [Ar] 3d 4s 4p
10 2 3
51Sb= [Kr] 4d 5s 5p
14 10 2 3
83Bi = [Xe] 4f 5d 6s 6p

 The first two elements of this group, nitrogen and phosphorous are non-metals, the next two,
arsenic and antimony are metalloids whereas the last element, bismuth is a metal.
 All the elements of this group have five electrons in their valence shells as ns2np3.
 Atomic radii is expected, increase with increase in atomic number. N<P<As<Sb<Bi

Atomic Structure & Basics of Chemistry Page 37


 The first ionization energy decreases regularly down the group due to increase in size of
atoms. N>P>As>Sb>Bi
 The ionization energies of this group are much higher than those of corresponding elements of
group 14.
N>C; P>Si; As>Ge; Sb>Sn; Bi>Pb
This is due to the increase in nuclear charge and extra stable configuration (i.e. half filled p-
orbitals) of the elements of this group.
 Electropositive (or metallic) character increase on going down the group due to decrease in
ionization energies.
 The value for electronegativity also decreases gradually with increase in atomic number.

2. Oxidation states
(a) Positive Oxidation States: The outer shell electronic configuration of these elements ns2np3
hence they can have oxidation states of +3 and +5. In addition to +3 and +5 states, nitrogen
also shows +1, +2 and +4 states.
(b) Negative oxidation States: All these elements (except Bi) show -3 oxidation state e.g. in
NH3 molecule, nitrogen shows -3 oxidation state. Nitrogen also shows -1 and -2 oxidation
state in NH2OH and N2H4

3.6.4 Oxygen family (Elements of group VIA)


The VIA sub group of the periodic table contains five elements viz. Oxygen (O), Sulphur (S),
Selenium (Se), Tellurium (Te) and Polonium (Po).
1. Electronic configuration: The electronic configurations of these elements are given below:
2 4
8O = [He] 2s 2p
2 4
16S = [Ne] 3s 3p
10 2 4
34Se = [Ar] 3d 4s 4p
10 2 4
52Te = [Kr] 4d 5s 5p
14 10 2 4
84Po = [Xe] 4f 5d 6s 6p

From the above electronic configurations it is clear that these elements have ns2np4 electrons in
their outermost shell.
2. Similarities and Gradation in Physical properties
 Metallic character increases gradually from oxygen to polonium. Oxygen and sulphur are
non-metals; selenium and tellurium are metalloids while the polonium is a radioactive metal.
 Oxygen has very high ionization energy than other elements of this group, i.e. the decrease
ionization energy from oxygen to polonium is due to increase in size of the atoms and greater
metallic character down the group.

Atomic Structure & Basics of Chemistry Page 38


Ionization energies of this group elements have lower than the corresponding 15th group
elements due to incompletely filled electronic configurations. i.e. O<N; S<P; Se<As; Te<Sb
and Po<Bi.
 The value of electronegativity decreases gradually down the group due to decrease in
effective nuclear charge.
 Oxidation states: These group elements have ns2np4 electrons in their outer shell, and they
can attain noble gas configuration either by gaining or by sharing two electrons. Thus, these
elements show two types of oxidation states.
(a) Negative oxidation states: Oxygen gives -2 oxid. states (except OF2).

3.7 IIA Group elements (Alkaline earth metals)


1. Oxides and hydroxides
Beryllium oxide, BeO, is formed by ignition of Be or its compounds in O2. It is an insoluble
white solid. The oxides of other group.2 metals are usually prepared by thermal decomposition
of the corresponding carbonate. MCO3 MO + CO2
The oxides of IIA group metal are ionic, so they react with water to produce the hydroxides.
MO + H2O  M(OH)2
However, beryllium oxide is quite different, and it exhibits amphoteric behavior.
Be (OH)2 + 2 OH- Be(OH)42-
Be (OH)2 + 2 H+  Be2+ + 2H2O
Magnesium hydroxide is a week, almost insoluble base that is used as a suspension known as
“milk of magnesia’ that is a common antacid. Calcium hydroxide is produced in enormous
quantities by the decomposition of limestone.
CaCO3  CaO + CO2
This is followed by the reaction of CaO (lime) with water. CaO + H2O  Ca(OH)2
Calcium hydroxide is known as hydrated lime or slaked lime, and it is used extensively in
some applications because it is less expensive than NaOH or KOH. It reacts with CO2 to form
CaCO3, which binds particles of sand and gravel together in mortar and cement.

Atomic Structure & Basics of Chemistry Page 39


Chapter - IV Molecular Geometry

4.1 Molecular Geometry


Molecular geometry is the three dimensional arrangement of atoms in a molecule. Molecules
geometry affects its physical and chemical properties. The geometry of the molecule ultimately
assumes minimizes the repulsion. This approach to the study of molecular geometry is called the
valence-shell electron pair repulsion (VSEPR) model. With this model, we can predict the
geometry of molecules (and ions) in a systematic way. For this purpose, it is convenient to divide
into two categories, according to whether or not the central atom has lone pairs.
1. Molecules in which the central atom has no lone pairs
For our convenient we will consider a molecule that contains atoms of only two elements of
electron pairs around the central atom A. The following table shows five possible arrangements
of electron pairs around the central atom A. Molecules in which the central atom has no lone
pairs have one of these five arrangements of bonding pairs. Let us take a close look at the
geometry of molecules with the formula AB2, AB3, AB4, AB5, and AB6.
Table.1 Geometry of simple molecules and ions in which the central atom has no lone pairs.
------------------------------------------------------------------------------------------------------------
Number of Type of Arrangement Molecular Examples
Electron pairs molecule electron pairs Geometry
------------------------------------------------------------------------------------------------------------
2 AB2 : ------ A ----- : B-----A------A BeCl2, HgCl2, BeF2,

3 AB3 BF3

4 AB4 CH4, NH4+

5 AB5 PCl5

6 AB6 SF6

Atomic Structure & Basics of Chemistry Page 40


2. Molecules in which the central atom has one or more lone pairs
Determination the geometry of a molecule is more complicated if the central atom has
both lone pairs and bonding pairs. In general, according to the VSEPR model, the repulsive
forces decrease in the following order:
Lone-pair vs lone-pair repulsion  lone-pair vs bonding pair repulsion  bonding pair vs
bonding pair repulsion
According to this,
 We designate molecules with lone pairs as ABxEy, where A is the central atom, B is a
surrounding atom and E is a lone pair on A. Both x and y are integers; x= 2,3,….. and
y=1,2,3,…….
 The simplest such molecule would be a triatomic molecule with one lone pair on the
central atom and the formula is AB2E. The following table shows, the geometry of
molecules whether the presence of lone pairs on the central atom.
Table.2 Geometry of molecules & ions in which the central atom has one or more lone pairs
---------------------------------------------------------------------------------------------------------------------
Class of Total no. of No. of bond No. of lone Geometry
-
molecule e pairs pairs pairs
---------------------------------------------------------------------------------------------------------------------
AB2E 3 2 1 Bent

AB3E 4 3 1 Trigonal
Pyramidal
AB2E2 4 2 2 Bent

AB4E 5 4 1 Distorted
Tetrahedron
AB3E2 5 3 2 T-shaped

AB2E3 5 2 3 Linear

AB5E 6 5 1 Square
Pyramidal
AB4E2 6 4 2 Square planar

---------------------------------------------------------------------------------------------------------------------

Atomic Structure & Basics of Chemistry Page 41


4.2 Dipole moments
The HF is a covalent molecule with a polar bond. There is a shift of electron density from H to F
because the F atom is more electronegative than H atom. The shift of electron density is
symbolized as a crossed arrow () above the Lewis structure to indicate the direction of the
shift. For example H – F: + -
The charge separation can be represented as, H – F
Where  (delta) denotes a partial charge
 The polarity of a bond measured as dipole moment () by the product of the charge (Q)
and the distance r between the charges,  = Q x r
 The  is always positive, usually expressed in debye units (D). The conversion factor is
1D=3.336 x 10-30 C.m. Here, C is coulomb and m is meter.
 Diatomic molecules containing atoms of different elements (HCl, CO and NO) have
dipole moments and are called polar molecules.
 Diatomic molecules containing atoms of the same element (H2, O2 and F2) are non polar
molecules because they do not have dipole moments.
 Carbon dioxide (CO2) is a triatomic molecule, so its geometry is either linear or bent;
O=C=O C
Linear molecule Resultant dipole moment
(no dipole moment) (has a dipole moment)
 Experimentally it is found that CO2 has no dipole moment (DM). Therefore, we conclude
that the CO2 molecule is linear.
Let us consider the NH3 and NF3 molecule
In both cases the central N atom has a lone pair, whose charge density is away from
the N atom. We know that N is more electronegative than H and F is more electronegative
than N. For this reason, the shift of electron density in NH3 is toward N and so contributes a
larger dipole moment (DM), where as the NF bond moment are directed away from N atom
and so the contribution of the lone pair to the dipole moment. Thus, the resultant DM in NH3
is larger than that in NF3.
 DM’s can be used to distinguish between molecules that have the same formula but
different structures (called isomers). For example, the following molecules both have the
same molecular formula (C2H2Cl2), but different molecular structures:
Because cis-dichloro ethylene is a polar molecule but trans-dichloro ethylene is not, they
can readily be distinguished by a dipole moment measurement.

Atomic Structure & Basics of Chemistry Page 42


4.3 Valence Bond Theory
 The VSEPR model provides a relatively simple and straight forward method for
predicting the geometry of molecules.
 For a more complete explanation of a chemical bond, we look to quantum mechanics.
Infact, the quantum mechanical study of chemical bonding also provides a means for
understanding the molecular geometry.
 Two quantum mechanical theories are used to describe covalent bond formation and the
electronic structure of molecules, such as valence bond (VB) theory and molecular orbital
(MO) theory.
 We discuss the VB theory by considering the formation of H2 molecule from two H
atoms. H – H  H2 molecule
(atoms)
By overlap of two 1s atomic orbitals, H-H bond formed. It means, two atomic orbitals
share a common region in space. i.e.
 In beginning the potential energy of two H atoms is zero. As the atoms approach each
other, each electron is attracted by the nucleus of the other atom. So, their potential
energy decreases (it becomes negative) as the atoms approach each other, shown in figure
(as follows),
 This trend continues until the potential energy reaches a minimum value, which has the
lowest potential energy, it is most stable. This condition corresponds to substantial
overlap of the 1s orbital and the formation of a stable H2 molecule.
 Valence bond theory states that a stable molecule forms from reacting atoms when the
potential energy of the system has decreased to a minimum.

Atomic Structure & Basics of Chemistry Page 43


Chapter. V Chemical Reactions

5.1 Chemical Equations


Process in which, a substance is changed into one or more new substances. To communicate with
one another about chemical reactions, chemists have devised a standard way to represent those
using chemical equations. In chemical equation, chemical symbols are used to show what
happens during a chemical reaction.
5.1.1 Writing chemical equations
Consider, when hydrogen gas (H2) burns in air (O2) to form water (H2O). This reaction can be
represented by the chemical equation. H2 + O2 H2O
Where, the plus sign means “reacts with” and the arrow means, “to yield”. Thus, this symbolic
expression can be read, “molecular hydrogen reacts with molecular oxygen to yield water”. The
above said reaction is not complete. So, we can balance the equation by balancing the
appropriate coefficient in front of H2 and H2O as, 2H2 + O2 2H2O
This balanced chemical equation shows that “two hydrogen molecules can combine or react with
one oxygen molecule to form two water molecules”, which is can also be read as, “2 moles of
hydrogen molecules react with 1 mole of oxygen molecules to produce 2 moles of water
molecules”. In terms of mass the equation is expressed as 4.04g of H2 react with 32g of O2 to
give 36.04g of H2O.
In the above equation, H2 and O2 are reactants, which are starting materials in a chemical
reaction. Water is the product which is the substance formed as a result of a chemical reaction. In
a chemical equation the reactants are written on the left and the products on the right of the
arrow. Reactants Products
Provide the additional information, chemists often indicate the physical states of the reactants
and products by using the letters g, l and s, denote gas, liquid and solid respectively.
2CO (g) + O2 (g) 2CO2 (g)
2HgO (s) Hg(l) + O2 (g)
The states of the reactants and products are especially useful in the laboratory. For example,
when potassium bromide (KBr) and silver nitrate (AgNO3) react in an aqueous environment, a
solid, silver bromide (AgBr) is formed. This reaction can be represented by the equation,
KBr (aq) + AgNO3 (aq) KNO3 (aq) + AgBr (s)
5.2 The mole concept
A mole is defined as the amount of substance that contains as many elementary entities (atoms,
molecules or other particles) in exactly 12g of carbon-12.
 Avogadro’s number can also determine the actual number of atoms experimentally. Thus, one
mole of carbon-12 contains 6.023x1023 atoms.
 The molar mass of a compound (in grams) is numerically equal to its molecular mass (in
amu). For example the molecular mass of water is 18.02amu so, its molar mass is 18.02g.
Thus, 1 mole of water weights 18.02g contains 6.023x1023 H2O molecule.

Atomic Structure & Basics of Chemistry Page 44


The following examples show to calculate the numbers of moles and individual atoms in a given
quantity of a compound as,
Example.1 How many moles of CHCl3 are in 198g of CHCl3?
Solution: molar mass of CHCl3 = 119.4g
1 mol of CHCl3 = 119.4g of CHCl3
so, 198g CHCl3 = ? mol of CHCl3
1 mol CHCl3
-------------------- x 198g CHCl3 = 1.66 mol CHCl3
119.4g CHCl3
Example.2 How many moles of He atoms are in 6.46g of He?
Solution: molar mass of He = 4.003g
1 mol of CHCl3 = 4.003g of He
so, 6.46g of He = ? mol of He
1 mol He
-------------------- x 6.46g He = 1.61 mol CHCl
4.003g He
Example.3 How many grams of Na2CO3 are present in 3.65mol of Na2CO3.
Solution: Molar mass of Na2CO3 = 58.47 g
1 mol Na2CO3 = 58.47g Na2CO3
3.65 mol Na2CO3 = ?
58.47g Na2CO3
------------------- x 3.65 mol Na2CO3 = 213.42g Na2CO3
1 mol Na2CO3
Example.4 Methanol burns in air according to the equation,
2CH3OH + 3O2 2CO2 + 4 H2O
If 209g of methanol are used up in a combustion process, what is the mass of H2O
produced.
Solution: molar mass of CH3OH = 32g
molar mass of H2O = 18g
32g of CH3OH produced 2 mol of H2O ---36g H2O
209g of CH3OH ---------- ?
36/32 x 209g CH3OH = 235.125g of H2O
Example.5 How many hydrogen atoms are present in 25.6g of urea, [(NH2)2CO]
1 mol of urea = 60.06g of urea
25.6g of urea = ? g of H atoms
1 mol urea 6.023x1023 molecules urea 4 H atoms
Number of H atoms = -------------- x 25.6g urea x ---------------------------------- x --------------------
60.06g urea 1 mol urea 1molecule urea

= 1.03x1024H atoms
Atomic Structure & Basics of Chemistry Page 45
Example.6 How many Cl atoms are present in 122g of MgCl
Solution: Molar mass of MgCl = 59.76g
1 mol MgCl 6.023x1023 molecules urea 1 Cl atom
Number of H atoms = -------------- x 122g MgCl x ---------------------------------- x ------------------
59.76g MgCl 1 mol MgCl 1molecule MgCl
= 1.23x1024 Cl atoms
5.3 Balancing of the chemical equations
In general, we balance chemical equations by the following steps,
 Identify all reactants and products and write their correct formulas on each sides of the
equation.
 Start to balance the equation by trying different coefficients on both sides of the equation. We
can change the coefficients but not the subscripts.
 Look the formulas containing the elements must have the same coefficient on each side, there
is no need to adjust the coefficient of these element at this point. Next, look for elements that
appear only once on each side of the equation but in unequal numbers of atoms, balance these
elements. Finally, balance the elements that appear in two or more formulas on the same side
of the equation.
 Check your balanced equation to be sure that have the same total number of each type of
atoms on both sides of the equation.
Let us consider, small amounts of oxygen gas can be prepared by heating potassium chlorate
(KClO3). The products are O2 and KCl. So, their respective equation is,
KClO3 KCl + O2
All these elements appear only once on each side of the equation, but only for K and Cl have
equal number of atoms on both sides. Thus, KClO3 and KCl must have the same coefficient. The
next step is to make the number of oxygen atoms the same on both sides of equation. We can
balance the O atoms by placing a 2 in front of KClO3 and a 3 in front of O2. We get,
2 KClO3 KCl + 3O2

Finally, we balance the K and Cl atoms by placing a 2 in front of KCl. So,


2KClO3 2KCl + 3O2
 At a final check, we can draw up balance the reactants and products,
Reactants Products
K (2) K (2)
Cl (2) Cl (2)
O (6) O (6)
Example.2 Balance the following chemical equation, C2H6 + O2 CO2 + H2O
Solution: Look at the coefficients of C,H and O on both side and change the coefficients of those
and get the balanced equation as,
C2H6 + 7/2 O2 2CO2 + 3H2O

Atomic Structure & Basics of Chemistry Page 46


However, we normally prefer to express the coefficients as whole numbers rather than fraction.
Therefore, we multiply the entire equation by 2 to convert 7/2 to, and we get fully balanced
equation as,
2 C2H6 + 7O2 4CO2 + 6H2O
We follow the above said method of balancing the chemical equation, will do the
balancing of some important examples of chemical reaction equations as,
Balance the following chemical equations.
1. Al + O2 Al2O3
Al and O appear only once on each side with unequal numbers,
2Al + 3/2O2 Al2O3
As like earlier case, we multiply the entire equation by 2, so,
4Al + 3O2 2Al2O3
2. Li + H2O LiOH + H2
Balanced equation for this, 2Li + 2H2O 2LiOH + H2
3. HCl + CaCO3 CaCl2 + H2O + CO2
Balanced equation for this, 2HCl + CaCO3 CaCl2 + H2O + CO2
4. NaOH + H2SO4 Na2SO4 + H2O
Balanced equation for this, 2NaOH + H2SO4 Na2SO4 + 2H2O
5. NaHCO3 Na2CO3 + H2O + CO2
Balanced equation for this, 2NaHCO3 Na2CO3 + H2O + CO2
6. Fe2O3 + CO Fe + CO2
Balanced equation for this, Fe2O3 + 3CO 2Fe + 3CO2

5.4 Amount of reactants and products


Stoichiometry is the quantitative study of reactants and products in a chemical reaction. Whether
the units given for reactants (or products) are moles, grams, liters or some other units, we use
moles to calculate the amount of product formed in a reaction. This approach is called the mole
method, which means simply that the stoichiometric coefficients in a chemical equation can be
interpreted as the number of moles of each substance. For example the Haber synthesis of
ammonia production as,
N2 + 3H2 2NH3
We read this equation as “1 mole of N2 gas combine with 3 moles of hydrogen gas to
form 2 moles of NH3.
Based on the mole method 3 types of stoichiometric calculations are followed as,
 Moles of reactant moles of product
 Mass of reactant moles of reactant moles of product
 Mass of reactant moles of reactant moles of product mass of product

Atomic Structure & Basics of Chemistry Page 47


The mole method consist the following steps,
 Write correct formulas for all reactants and products and balance the resulting equation.
 Convert the known quantities of given substances (reactants) into moles
 Use the coefficients in the balanced equation to calculate the number of moles of unknown
quantities (products)
 Convert the unknown quantities to whatever units are required (typically grams)
 Check that your answer is reasonable in physical terms.
In stoichiometry we use the symbol , which means “stoichiometrically equivalent to” or simply
“equivalent to”. In the case of balanced equation for the ammonia synthesis, 1 mol of N2 reacts
with 3 mol of hydrogen. So, 1 mole of N2 is equivalent to 3 moles of H2 as,
1 mol of N2  3 mol of H2
We can write the unit factor as,
1 mol of N2 3 mol of H2
-------------- = -------------------
3 mol of H2 1 mol of N2
The following examples illustrate the use of the five-step method in solving some typical
stoichiometry problems.
Example.1 All alkali metals react with water to produce hydrogen gas and the corresponding
alkali metal hydroxide. A typical reaction is that between Li and H2O.
2Li (s) + 2H2O (l) 2LiOH (aq) + H2 (g)
(a) How many moles of H2 will be formed by the complete reaction of 6.23 moles of Li with
H2O.
(b) How many grams of H2 will be formed by the complete reaction of 80.57g of Li with water.
Solution: (a) The balanced equation is given in the problem
No conversion is needed, because the amount of starting material, Li is given in moles.
Since 2 moles Li produce 1 mol H2 or 2 mol Li  1 mol H2, we calculate moles of H2
produced as follows,
moles of H2 produced = 1 mol H2 / 2 mol Li x 6.23 mol Li
= 3.12 mol H2
Check our answer started with 6.23 moles of Li and produced 3.12 moles of H2.
Since 2 moles of Li produce 1 mol of H2, 3.12 moles is reasonable quantity.
(b) The reaction is same as earlier.
The number moles of Li = 80.57g Li x 1 mol Li/ 6.941 g Li = 11.61 mol Li
2 mol Li  1 mol H2, we calculate the number of moles of H2 as follows,
moles of H2 produced = 1 mol H2 / 2 mol Li x 11.61 mol Li = 5.805 mol H2
From the molar mass of H2 (2.016g) we calculate the mass of H2 produced

Atomic Structure & Basics of Chemistry Page 48


Mass of H2 produced = 2.016g H2 / 1 mol H2 x 5.805 mol H2 = 11.70 g H2
Check our answer – Because the molar mass of H2 is smaller than that of Li and two moles of
Li are needed to produce one mole of H2, we expect the answer to be smaller than 80.57g.
Example.2 The reaction between nitric oxide (NO) and oxygen to form nitrogen dioxide (NO2)
is a key step in photochemical smog formation.
2NO(g) + O2(g) 2NO2(g)
(a) How many moles of NO2 formed by the complete reaction of 0.254 mole of O2?
(b) How many grams of NO2 formed by the complete reaction of 1.44g of NO?
Solution (a) According to our balanced equation, 1 mol O2  2 mol NO2
Moles of NO2 formed = 2 mol NO2 / 1 mol O2 x 0.254 mol O2
= 0.508 mol NO2 produced
Since 1 mol of O2 produced 2 mol NO2. So, we expect 0.254 mol O2 produce 0.508 mol of NO2
as doubled is reasonable.
(b) We convert 1.44g of NO into moles.
So, mol NO=1 mol NO/30g NO x 1.44g NO = 0.048mol NO
2 mol NO produce 2 mol NO2 i.e., 2 mol NO  2 mol NO2
so, 0.048 mol NO produced 0.048 mol NO2
Mass of NO2 produced = 46g NO2 / 1 mol NO2 x 0.048 mol NO2 = 2.208g NO2
Because the molar mass of NO2 is larger than NO and two moles of NO produce two moles
of NO2, we expect the answer to be larger than 1.44g.
Example.3 In the case of Haber process of NH3 production, nitrogen react with hydrogen gas as,
N2 + 3H2 2NH3
Work out the followings,
(a) How many moles of NH3 formed by the complete reaction of 6.38 moles of N2?
(b) How many moles of H2 used in this reaction?
(c) How many grams of N2 used for the production of 114g of NH3?
Solution
(a) 1 mol N2  2 mol NH3
so, 6.38 mol N2 produced 12.76 mol NH3.
(b) 1 mol NH3 = 17 g of NH3
208g NH3 contains = 1 mol NH3 / 17g NH3 x 208g NH3
= 12.24 mol NH3
According to balanced equation, 3 mol H2 used for the production of 2 mol of NH3
3 mol of H2  2 mol NH3
No of moles of H2 used for production of 12.24 mol NH3 = 3 mol H2/2 mol NH3 x 12.24 mol
NH3 = 18.36 mol H2

Atomic Structure & Basics of Chemistry Page 49


(c) 1 mol NH3 = 17 g of NH3
114g NH3 contains = 1 mol NH3 / 17g NH3 x 114g NH3
= 6.71 mol NH3
According to balanced equation, 1 mol. N2 used for the production of 2 mol of NH3
1 mol of N2  2 mol NH3
No of moles of N2 used for production of 6.71 mol NH3 = 1mol N2/2 mol NH3 x 6.71 mol
NH3
= 3.36 mol N2
1 mol of N2 = 28g of N2
so, 3.36mol N2 = 28g N2 / 1 mol N2 x 3.36 mol N2
= 94.08g of N2 used.
--------------------……………….----------------------
5.5 Limiting Reagents
Reactants are added to a reaction vessel in different amounts given by the chemical equation. In
such cases only one of the reactants may be completely consumed at the end of the reaction,
called limiting reagents. Whereas some amounts of other reactants will remain unreacted (not
completely consumed) is often referred as an excess reagent.
For example, sulfur hexafluoride (SF6) is formed by burning sulfur in an atmosphere of fluorine
as S (l) + 3F2 (g) SF6 (g)
This reaction says that 1mol of S reacts with 3 moles of F2 to produce 1 mol SF6. Suppose that 4
moles of S are added to 20 moles of F2.
Since 1 mol S  3 mol F2, the number of moles of F2 needed to react with 4 moles of S is,
4 mol S x 3 mol F2 / 1 mol S = 12 mol F2
But there are 20 moles of F2 available, more than needed to completely react with S. Thus
S must be the limiting reagent and F2 the excess reagent. The amount of SF6 produced depends
only on how much S was originally present.
Exercise.1 Urea is prepared by reacting NH3 with CO2, 2NH3 (g) + CO2 (g) (NH2)2CO (aq)
+ H2O(l)
In one process, 637.2g of NH3 are allowed to react with 1142g of CO2. (a) Which of two
reactants is the limiting reagent? (b) Calculate the mass of (NH2)2CO formed. (c) How much of
the excess reagent (in grams) is left at the end of the reaction?
Solution: (a) Molar mass of NH3 and CO2 are 17.03g and 44.01g
mol of NH3 = 1 mol NH3 / 17.03g NH3 x 637.2g NH3 = 37.42 mol NH3
mol of CO2 = 1 mol CO2 / 44.01g NH3 x 1142g CO2 = 25.95mol CO2
From the balanced equation, we see 2 mol NH3  1 mol CO2. So, number of moles of NH3
needed to react with 25.95 mol CO2 is, 2 mol NH3 / 1 mol CO2 x 25.95 mol CO2 = 51.90
mol NH3
Atomic Structure & Basics of Chemistry Page 50
Since there are only 37.42 mol of NH3 present, not enough to react completely with CO2. Hence
NH3 must be the limiting reagent and CO2 is an excess reagent.
(b) The amount of urea, (NH2)2CO produced is determined by the amount of limiting reagent
present, so
mass of (NH2)2CO = 1 mol urea/2mol NH3 x 37.42 mol NH3 x 60.06 g urea/ 1mol urea
= 1124g urea
(c) The number of moles of the excess reagent left at the end of the reaction is,
25.95 mol CO2 – [37.42 mol NH3 x 1 mol CO2 / 2mol NH3] = 7.24 mol CO2
and mass left over = 44.01g CO2 / 1 mol CO2 x 7.24 mol CO2
= 319 g CO2
5.6 Reaction Yield
Theoretical yield: The amount of limiting reagent present at the beginning of reaction
determines the theoretical yield of the reaction. It is the maximum obtainable yield, predicted by
the balanced equation.
Actual yield: In practically, the actual yield or the amount of product actually obtained from a
reaction is almost always less than the theoretical yield.
Percent yield: It is described as the proportion of the actual yield to the theoretical yield. It is
calculated as, actual yield
% yield = --------------------------- x 100 %
Theoretical yield
Percent yield ranged from a fraction of 1-100%

Exercise.1 Urea is prepared by reacting NH3 with CO2, 2NH3 (g) + CO2 (g) (NH2)2CO (aq) +
H2O (l)
In one process, 637.2g of NH3 are allowed to react with 1142g of CO2. (a) Which of two
reactants is the limiting reagent? (b) Calculate the theoretical yield of (NH2)2CO in grams (c)
Calculate the percent yield if 640g of urea actually obtained.
Solution: (a) calculate the number of moles of NH3 and CO2 initially present from their molar
mass of NH3 and CO2 are 17.03g and 44.01g respectively.
mol of NH3 = 1 mol NH3 / 17.03g NH3 x 637.2g NH3 = 37.42 mol NH3
mol of CO2 = 1 mol CO2 / 44.01g NH3 x 1142g CO2 = 25.95mol CO2

From the balanced equation, we see 2 mol NH3  1 mol CO2. So, number of moles of
NH3 needed to react with 25.95 mol CO2 is, 2 mol NH3 / 1 mol CO2 x 25.95 mol CO2 = 51.90
mol NH3. Since there are only 37.42 mol of NH3 present, not enough to react completely with
CO2. Hence NH3 must be the limiting reagent and CO2 is an excess reagent.
(b) From the amount of limiting reagent we can calculate the theoretical yield of urea, (NH2)2CO
as,

Atomic Structure & Basics of Chemistry Page 51


From the balanced equation, 2 mol NH3  1 mol urea
Theoretical yield = 1 mol urea/2mol NH3 x 37.42 mol NH3 x 60.06 g urea/ 1mol urea
= 1124g urea
(c) Therefore, % yield = actual yield / Theoretical yield x 100%
= 640g / 1124g x 100%
= 56.94%
Example.2 Titanium is a protective material, which is used in aircraft, spaces, bicycle spare parts
etc., Titanium is prepared by the reaction of TiCl4 with molten magnesium between 9500C and
11500C.
TiCl4 (g) + 2Mg (l) Ti (s) + 2MgCl (l)
In industrial operation 3.54x10 g of TiCl4 reacted with 1.13x107g Mg (a) Calculate the
7

theoretical yield of Ti in grams. (b) Calculate the percent yield if 7.91x106 g of Ti is actually
obtained.
Solution: First we find out limiting reagent, from this to calculate the theoretical yield.
(a) Calculate the number of moles of TiCl4 and Mg initially present.
moles of TiCl4 = 3.54x107g TiCl4 x 1 mol TiCl4 / 189.7g TiCl4 = 1.87x105 mol TiCl4
moles of Mg = 1.13x107g Mg x 1mol Mg / 24.31g Mg = 4.65x105 mol Mg
Next, we must determine which of the two substances is the limiting reagent. From the balanced
equation, 1 mol TiCl4  2 mol Mg; therefore the number of moles of Mg needed to react with
1.87x105 moles of TiCl4 is,
1.87x107 mol TiCl4 x 2 mol Mg/1 mol TiCl4 = 3.74x105 mol Mg
Since 4.65x105 mol of Mg are present more than is needed to react with the amount of TiCl4. So,
Mg must be the excess reagent and TiCl4 is the limiting reagent.
The equation shows that, 1 mol TiCl4  1 mol Ti ; thus the theoretical mass of Ti formed is,
1.87x105 mol TiCl4 x 1 mol Ti/1 mol TiCl4 x 47.88g Ti/ 1 mol Ti = 8.95x106g Ti
(b) The % yield of this reaction,
% yield = actual yield / Theoretical yield x 100%
= 7.91x106g / 8.93x106g x 100% = 88.6%
Example.3 Industrially, vanadium metal, which is used in steel alloys, can be obtained by
reacting vanadium(V) oxide with calcium at high temperature.
5Ca + V2O5 5CaO + 2V
In one process 1540g of V2O5 react with 1960g of Ca. (a) Calculate the theoretical yield of V. (b)
Calculate the percent yield if 803g of V are obtained.
Solution: (a) Calculate the number of moles of V2O5 and Ca initially present,
Moles of V2O5 = 1 mol V2O5 / 181.88g V2O5 x 1540g V2O5 = 8.46 mol V2O5
Moles of Ca = 1 molCa/40.08g Ca x 1960g Ca = 48.9 mol Ca

Atomic Structure & Basics of Chemistry Page 52


We must determine the limiting reagents. From the balanced equation 5 mol Ca  1mol V2O5;
therefore the number of moles of Ca needed to react with 8.46 mol V2O5,
= 5 mol Ca/1 mol V2O5 x 8.46 mol V2O5 = 42.3 mol Ca
But we have 48.9 mol Ca is more than needed to react with V2O5. So, we conclude that Ca is an
excess reagent and V2O5 is a limiting reagent.
The equation shows, 1molV2O5  2 mol V. Thus, theoretical mass of V formed is,
8.46 mol V2O5 x 2mol V/1 mol V2O5 x 50.94g V/1 mol V = 862.63 g V
(b) The % yield of this reaction = actually yield / Theoretical yield x 100%
= 803g/862.33 x 100%
= 93.12%
---------------------------------………………….-------------------------
5.7 Precipitation Reactions
One common type of reaction occurs in aqueous solution is the precipitation reaction, which
results in the formation of an insoluble solid that separates from the solution. Precipitation
reactions usually involve ionic compounds.
For example, aqueous lead nitrate (Pb(NO3)2 reacts with aqueous sodium iodide (NaI) produce
insoluble yellow precipitate lead iodide (PbI2), sodium nitrate remains in solution as,
Pb(NO3)2 (aq) + 2NaI (aq) ------------ PbI2 (s) + 2NaNO3 (aq)

Yellow in solution
Precipitate
5.7.1 Solubility
How can we predict whether a precipitate will form when a compound is added to a solution or
when two solutions are mixed? It depends on the solubility of the solute, which is defined as the
maximum amount of solute that will dissolve in a given quantity of solvent at a specific
temperature.
A substance is said to be soluble if a fair amount of it visibly dissolves when added to water. If
not, the substance is described as slightly soluble or insoluble. All ionic compounds are strong
electrolytes, but they are not equally soluble. Following table classifies a number of common
ionic compounds as soluble or insoluble.

Atomic Structure & Basics of Chemistry Page 53


Table: Solubility rules for common ionic compounds in H2O at 250C

Soluble compounds Exceptions

Compounds containing alkali metal ions (Li+, Na+,K+,Rb+, Nitrates (NO3-) bicarbonates
Cs+) and the ammonium ion (NH4+). (HCO3-) and Chlorates (ClO3-)
Halides (Cl-, Br-,I-) Halides of Ag+, Hg22+ and Pb2+
Sulfates (SO42-) Sulfates of
Ag+,Ca2+,Sr2+,Ba2+,Hg2+& Pb2

Insoluble compounds Exceptions

Phosphates (PO43-), Chromates (CrO42-) Compounds containing alkali metal


Sulfides (S2-) ions and ammonium ion.

Hydroxides (OH-) Compounds containing alkali


metal ions andBa2+

Exercise.1 Classify the following compounds as soluble or insoluble (a) Silver sulfate (Ag2SO4)
(b) Calcium carbonate (CaCO3) and (c) Sodium phosphate (Na3PO4)
Solution: According to solubility rules,
(a) Ag2SO4 is insoluble
(b) This is a carbonate and Ca is a IIA group metal. Therefore, CaCO3 is insoluble.
(c) Sodium is an alkali metal (groupIA). So, Na3PO4 is soluble.
Exercise.2 Classify the following ionic compounds as soluble or insoluble: (a) CuS (b) Ca(OH)2
and (c) Zn (NO3)2
Solution: According to solubility rules,
(a) CuS is insoluble
(b) Ca(OH)2 is a hydroxide and Ca is a IIA group metal. Therefore, Ca(OH)2 is insoluble in
water.
(c) Zn(NO3)2 is a nitrates, which is soluble in water.

-------------------------------……………..---------------------------------

Atomic Structure & Basics of Chemistry Page 54


5.8 Molecular equations and Ionic equations
The equation describing the precipitation reaction is called a molecular equation. A molecular
equation is useful because it identifies the reagents, but does not accurately describe what
actually happened at the microscopic level. When ionic compounds dissolve in water, they break
apart into their component cations and anions. The equations should show the dissociation of
dissolved ionic compounds into ions. Therefore, returning to write the reaction between sodium
iodide and lead nitrate as,
Pb2+ (aq) + 2NO3- (aq) + 2Na+ (aq) + 2I- (aq) PbI2 (s) + 2Na+ (aq) + 2NO3- (aq)
The above equation is an example of an ionic equation, which shows dissolved species as free
ions.
An ionic equation includes spectator ions, or ions that are not involved in the overall
reaction, in this case the Na+ and NO3- ions. Spectator ions appear on both sides of the equation
and are unchanged in the chemical reaction. So they can be cancelled, we write the net ionic
equation, which shows only the species that actually take part in the reaction,
Pb2+ (aq) + 2I- (aq) PbI2 (s)
Look at another example, BaCl2(aq) + Na2SO4(aq) BaSO4 (s) + 2NaCl (aq)
(White
precipitate)
The ionic equation for the reaction is,
Ba2+ (aq) + 2Cl- (aq) + 2Na+ (aq) + SO42- (aq) BaSO4 (s) + 2Na+ (aq) + 2Cl- (aq)
Canceling the spectator ions (Na+ and Cl-) on both sides of the equation and net ionic equation,
Ba2+ (aq) + SO42- (aq) BaSO4 (s)

Exercise.1 Predict the products of the following reaction and write net ionic equation for the
reaction.
K3PO4 (aq) + Ca(NO3)2 (aq) ?
Solution
 Fix the charges on cations and anions, when these ionic compounds dissolve in water,
they
Produce the K+, PO43-,Ca2+ and NO3-. According to solubility rule Ca2+ and PO43- can form an
insoluble, calcium phosphate [Ca(PO4)2]. Therefore, this is a precipitation reaction. The other
product, KNO3 is soluble and remains in solution. The molecular reaction is,
2K3PO4 (aq) + 3Ca(NO3)2 (aq) KNO3 (aq) + Ca3(PO4)2 (s)
and the ionic equation for this is,
6K+ (aq) + 2PO43- (aq) + 3Ca2+ (aq) + 6NO3- (aq) Ca3(PO4)2 (s) + 6K+ (aq) + 6NO3- (aq)
Canceling the spectator ions and obtain the net ionic equation,
3Ca2+ (aq) + 2PO43- (aq) Ca3(PO4)2 (s)
----------------------………………….--------------------------

Atomic Structure & Basics of Chemistry Page 55


Practice Exercise.1 Predict the precipitate produced by the following reaction and write a net
equation for the reaction.
Al(NO3)3 (aq) + NaOH (aq) ?
Solution
They produce the Al3+, NO3-, Na+ and OH-, when dissolved in water. According to
solubility rules, Al3+ and OH- ions can form insoluble compound, Al(OH)3. Therefore this is a
precipitation reaction. The NaNO3 remains in solution. Therefore the molecular reaction is,
3Al(NO3)3 (aq) + 9NaOH (aq) ----------- 3Al(OH)3 (s) + 9NaNO3
The ionic equation,
3Al3+ (aq) + 9NO3- (aq) + 9Na+ (aq) + 9OH-(aq) 3Al(OH)3(s )+ 9Na+ (aq) + 9NO3-(aq)
Canceling the spectator ions, Na+ and NO3-, get net ionic equation as,
3Al3+ (aq) + 9OH- (aq) 3Al(OH)3 (s)

Some other exercises: Write net ionic equation for the followings,
1. 2AgNO3 (aq) + Na2SO4 (aq) ?
Balanced molecular reaction equation,
2AgNO3 (aq) + Na2SO4 (aq) Ag2SO4(s) + 2NaNO3 (aq)
Ionic equation, 2Ag (aq) +SO4 (aq) +2Na+(aq) +2NO3-(aq)
+ 2-
Ag2SO4 (s)+ 2Na+(aq) +2NO3-(aq)
Net ionic equation, 2Ag+(aq) +SO42-(aq) Ag2SO4 (s)
2. BaCl2 (aq) + ZnSO4 (aq) ?
Balanced molecular reaction equation, BaCl2 (aq) + ZnSO4 (aq) BaSO4(s) + ZnCl2 (aq)
2+ - 2+ 2-
Ionic equation, Ba (aq) +2Cl (aq) +Zn (aq) +SO4 (aq) BaSO4(s)+ Zn2+(aq) +2Cl-(aq)
Net ionic equation, Ba2+(aq) + SO42-(aq) BaSO4(s)
3. (NH4)2CO3 (aq) + CaCl2 (aq) ?
Balanced molecular reaction equation, (NH4)2CO3 (aq) + CaCl2 (aq) 2NH4Cl (aq) + CaCO3(s)
+ 2- 2+ -
Ionic equation, 2NH4 (aq) + CO3 (aq) + Ca (aq) + 2Cl (aq) CaCO3(s) + 2NH4+(aq) + 2Cl-(aq)
Net ionic equation, Ca2+(aq) + CO32-(aq) CaCO3(s)
4. Na2S (aq) + ZnCl2 (aq) ?
Balanced molecular reaction equation, Na2S (aq) + ZnCl2 (aq) ZnS (s) + 2 NaCl(aq)
+ 2- 2+ -
Ionic equation, 2Na (aq) + S (aq) + Zn (aq) + 2Cl (aq) ZnS (s) + 2Na+(aq) + 2Cl-(aq)
Net ionic equation, Zn2+(aq) + S2-(aq) ZnS (s)
5. 2K3PO4 (aq) + 3Sr(NO3)2 (aq) ?
Balanced molecular reaction equation, 2K3PO4 (aq)+3Sr(NO3)2 (aq) Sr3(PO4)2(s) + 6KNO3(aq)
Ionic equation, 6K+(aq)+2PO43-(aq)+3Sr2+(aq)+6NO3-(aq) Sr3(PO4)2(s)+6K+(aq)+6NO3-(aq)
Net ionic equation, 3Sr2+(aq)+ 2PO43-(aq) Sr3(PO4)2(s)

Atomic Structure & Basics of Chemistry Page 56


6. Mg(NO3) 2 (aq) + 2NaOH (aq) ?
Balanced molecular reaction, Mg(NO3)2(aq)+2NaOH (aq) Mg (OH)2(s)+2NaNO3(aq)
Ionic equation, Mg2+(aq)+2NO3-(aq)+2Na+(aq)+2OH-(aq) Mg(OH)2(s) + 2Na+(aq) + 2NO3-(aq)
Net ionic equation, Mg2+(aq)+2OH-(aq) Mg(OH)2(s)

5.9 Acid-base neutralization


The neutralization reaction between acid and base produce water and a salt.
Acid + base salt + water
For example, HCl (aq) + NaOH (aq) NaC l(aq) + H2O (l)
The ionic equation,
H+(aq) + Cl-(aq)+Na+(aq) + OH-(aq) Na+(aq) + Cl- (aq) + H2O (l)
Therefore the reaction can be represented by the net ionic equation,
H+(aq)+OH- (aq) H2O (l)
+ -
Both Na and Cl are spectator ions.
5.10 Oxidation-Reduction Reactions
Acid-base reactions can be characterized as proton transfer processes, the class of
reactions called oxidation-reduction or redox reactions is considered as electron-transfer
reactions. Many important redox reactions take place in water, but not all redo reactions occur in
aqueous solution. Consider the formation of CaO from Ca and O2.
2Ca(S) + O2 (g) 2CaO (s)
Calcium oxide (CaO) is an ionic compound made up of Ca2+ and O2- ions.
In this reaction, 2Ca atoms give up or transfer four electrons to two ‘O’ atoms. For our
convenient this process separate into two steps, one involving the loss of 4 electrons and the
other being the gain of four electrons by an O2 molecule.
2Ca 2Ca2+ + 4e-
O2 + 4e- 2O2-
Each of these steps is called a half-reaction. The sum of the half-reactions gives the overall
reaction: 2Ca + O2 + 4e- 2Ca2+ + 2O2- + 4e-
so, 2Ca + O2 2Ca2+ + 2O2-
Finally the Ca2+ and O2- ions combine to form CaO as,
2Ca2+ + 2O2- 2CaO
 The term oxidation reaction refers to the half-reaction that involves loss of electrons.
 A reduction reaction is a half reaction that involves gain of electrons.
 In the formation of CaO, Ca is oxidized. If said to be act as a reducing agent, because it
donates electrons to oxygen and causes O2 to be reduced.

Atomic Structure & Basics of Chemistry Page 57


 O2 is reduced and acts as an oxidizing agent because it accepts electrons from calcium.
Another example of redox reaction,
Zn (s) + CuSO4 (aq) ZnSO4 (aq) + Cu (s)
2+ 2+
Ionic equation, Zn (s) + Cu (aq) Zn (aq) + Cu (s)
Oxidation and reduction half-reactions are,
Zn Zn2+ + 2e- (oxidation)
Cu2+ + 2e- Cu (reduction)
+
Similarly, metallic Cu reduces Ag in a solution of silver nitrate, AgNO3
Cu (s) + 2AgNO3 (aq) Cu(NO3)2 (aq) + 2Ag (s)
+ 2+
or Cu (s) + 2Ag (aq) Cu (aq) + 2Ag (s)
5.10.1 Oxidation Number
An atom’s oxidation number, also called oxidation state, signifies the number of charges
the atom would have in a molecule if electrons were transferred completely.
For example, let us consider the formation of HCl and SO2 as,
H2(g) + Cl2(g) ------------ 2HCl(g)
S(s) + O2(g) -------------- SO2(g)
The numbers above the element symbols are the oxidation numbers.
 Oxidation numbers enable us to identify elements that are oxidized and reduced at a
glance.
 The elements that show an increase in oxidation number (hydrogen and sulfur in above
example) are oxidized chlorine and oxygen is reduced.
 The HCl and SO2 are neutral in terms of charges, so charges must be cancel.
5.10.1.1 Assignment of oxidation numbers
 In free elements each atom has an oxidation number of zero. Thus each atom in H2, Br2,
Na, Be, K, Cl2, I2, O2 and P4 etc. has the same oxidation number, zero.
 For ions composed of only one atom (mono atomic ions ) the oxidation number is equal
to charge on the ion. Thus, Li+ has an oxidation number +1; for Ba2+ - +2; for Fe3+ - +3;
I- = -1; Al3+ = +3; O2- = -2 and so on.
 The oxidation number of oxygen in most of the compounds is –2 (e.g. MgO and H2O),
but in hydrogen peroxide (H2O2) and peroxide ion (O22-), it is –1.
 The oxidation number of hydrogen is +1, but in the case of hydrides like LiH, NaH,
CaH2, its oxidation number is –1.
 All the halogens have the oxidation number –1. But, when combined with oxygen, for
example in oxyacids and oxoanions, they have positive oxidation numbers.
For e.g. in KClO3, the oxidation number of chlorine is +5 and in NaOCl, the oxidation
number of chlorine is +1.

Atomic Structure & Basics of Chemistry Page 58


 In a neutral molecule, the sum of the oxidation numbers of all the atoms must be zero. In
a polyatomic ion, the sum of oxidation numbers of all the elements in the ion must be
equal to the net charge of the ion. For e.g., in NH4+, the oxidation number of N is –3 and
that H is +1. Thus the sum of the oxidation numbers is –3+4(+1) = +1, which is equal to
the net charge of the ion.
 Oxidation numbers do not have to be integers. For e.g., the oxidation number of ‘O’ in
the superoxide ion, O2- is –1/2.

Exercise.1 Assign the oxidation numbers to all the elements in the following compounds and
ion.
(a) Li2O : Li = +1 (b) HNO3 : H+ = +1
O2- = -2 O2- = -2 so, N = +5
(c) Cr2O72- (e) MnO4-
Sum of oxidation number = -2 sum of oxidation number
For O = -2 for O = -2
Cr = x Mn = x
Therefore, -2 = 2(x) + 7(-2) Therefore, -1 = 1x + 4(-2)
- 2 = 2x –14 -1+8 = x
2x = 12 x = +7
x = +6
Therefore, the oxidation number of Cr is +6. Therefore oxidation numner of Mn is +7.

(d) PF3
Sum of oxidation number = 0
For F = -1
P =x
So, 0 = 1x + 3(-1)
X = +3
Therefore, oxidation number of P is +3.

Exercise.2 Arrange the following species in order of increasing order of oxidation number of
the sulfur atom; H2S , S8, H2SO4, S2-, HS-1, SO2, SO3.
In H2S, the oxidation number of S is –2
In S8 =0
In H2SO4 = +6
In S2- = -2
-1
In HS = -2
In SO2 = +4
In SO3 = +6

So, the increasing order as, H2S / S2- / HS-1  S8  SO2  SO3 / H2SO4

Atomic Structure & Basics of Chemistry Page 59


5.10.2 Types of redox reactions
There are four types redox reactions, combination reactions, decomposition reactions,
disproportionation reactions and displacement reactions.
Combination reaction
A combination reaction may be represented by,
A + B ----------- C
Combination reactions are reactions in which two or more substances combine to form a single
product. For e.g. S(s) + O2(g) ------------- SO2(g)
H2(g) + Cl2(g) ------------ 2HCl(g)
3Mg(s) + N2(g) ------------ Mg3N2(s)
Decomposition reactions
This is the opposite of combination reactions. Specifically, a decomposition reaction is the
breakdown of a compound into two or more components.
C ------------- A + B
If A or B is an element, then the reaction is redox in nature. For e.g.,
2 HgO(s) ------------- 2Hg(l) + O2(g)
2 KClO3(s) ------------ 2KCl(s) + 3O2(g)
2 NaH(s) ---------------- 2 Na(s) + H2(g)
Disproportionation reaction
A special type of redox reaction is the disproportionation reaction. In this case, an element in one
oxidation state is simultaneously oxidized and reduced. One reactant in a disproportionation
reaction always contains an element that can have at least three oxidation states.
 The decomposition of H2O2 is an example of a disproportionation reaction,
2H2O2 (aq) -------------- 2H2O (l) + O2(g)
Here the oxidation number of oxygen in the reactant (-1) both increase to zero in O2 and
decreases to –2 in H2O.
 Another example, is the reaction between molecular chlorine and NaOH solution,
Cl2 + 2 OH-(aq) ----------- ClO-(aq) + Cl-(aq) + H2O(l)
This reaction describes the formation of household bleaching agents, for it is the
hypochlorite ion (ClO-).
Displacement reaction
In a displacement reaction, an ion ( or atom) in a compound is replaced by an ion ( or atom) of
another element.
i.e., A + BC ------------------ AC + B
Three subcategories are, hydrogen displacement, metal displacement and halogen displacement.
1. Hydrogen displacement reaction
All alkali metals and some alkaline earth metals (Ca, Sr & Ba), which are most reactive of
the metallic elements will displace hydrogen from cold water.
2 Na(s) + 2H2O(l) ---------- 2 NaOH (aq) + H2 (g)
Ca(s) + 2H2O (l) ------------ Ca(OH)2(aq) + H2 (g)
Atomic Structure & Basics of Chemistry Page 60
Less reactive metals, such as Al and Fe, react with steam to give hydrogen gas,
2 Al(s) + 3H2O(g) ------------- Al2O3 (s) + 3H2 (g)
2 Fe (s) + 3H2O(g) -------------- Fe2O3 (s) + 3H2 (g)
Many metals including those that do not react with water, are capable of displacing hydrogen
from acids. For example,
Zn(s) + 2HCl(aq) ---------------- ZnCl2 (aq) + H2 (g)
Mg (s) + 2HCl (aq) --------------- MgCl2 (aq) + H2 (g).
2. Metal displacement reactions
A metal in a compound can be displaced by another metal in the elemental state.
For example, vanadium is obtained by treating vanadium(V) oxide with metallic calcium.
V2O5 (s) + 5 Ca (l) -------------- 2V(l) + 5 CaO (s)
Similarly, titanium is obtained from TiCl4 according to the reaction,
TiCl4 (g) + 2 Mg (l) ------------ Ti (s) + 2 MgCl2 (aq)
An easy way to predict whether a metal or hydrogen displacement reaction will actually occur
is to refer to an activity series (sometimes called electrochemical series) shown in following
figure. Basically, an activity series is a convenient summary of the results of many possible
displacement reactions.

According to this series, any metal above hydrogen will displace it from water or from an acid,
but metals below hydrogen will not react with either water or an acid. In fact, any species listed
in the series will react with any species below it. For e.g., Zn is above Cu, so Zn metal will
displace Cu ions from copper sulphate as, Zn (s) + CuSO4 (aq) -------- ZnSO4 (aq) + Cu (s)

Atomic Structure & Basics of Chemistry Page 61


3. Halogen Displacement Reactions
The halogens behavior in halogen displacement reactions, F2  Cl2  Br2  I2
The power of these elements as oxidizing agents decreases as we move down from
fluorine
to iodine, so molecular fluorine can replace Cl-, Br- and I- ions in solution. The halogen
displacement equations are,
Cl2 (g) + 2 KBr (aq) ------------ 2KCl (aq) + Br2 (l)
Cl2 (g) + 2 NaI (aq) ------------ 2NaCl (aq) + I2 (l)
Molecular bromine can displace iodide ion in solution as,
Br2 (l) + 2 KI (aq) ------------- 2KBr (aq) + I2 (s)
Reversing the roles of the halogens produces no reaction. Thus, bromine cannot displace
chloride ions and iodine cannot displace bromide and chloride ions.

-----------------……………….-------------------
5.10.3 Balancing of Redox reactions
1. Balance the following redox reaction by using ion-electron method in basic medium.
MnO4- + I- MnO2 + I2
Divide into two half reactions
I- I2 (oxidation)
-
MnO4 MnO2 (reduction)
Balance the I atoms in the oxidation half reaction, 2I- I2
In the reduction half reaction, we add two H2O molecules on the right side to balance the O
atoms, MnO4- MnO2 + 2H2O
To balance the H atoms, we add 4H+ ions on the left side, we get,
MnO- + 4H+ MnO2 + 2H2O
Since the reaction occurs in basic medium and there are foir H+ ions we added. So, add
four OH- on both sides, MnO- + 4H+ + 4OH- MnO2 + 2H2O + 4OH-
Balance the charges of two half reactions, 2I- I2 + 2e-
MnO- + 4 H2O + 3e- MnO2 + 2H2O +
-
4OH
Equalize the number of electrons; we multiply the oxidation half reaction by 3 and reduction
half reaction by 2.
6I- 3I2 + 6e-
2MnO- + 4 H2O + 6e- 2MnO2 + 8OH-
----------------------------------------------------------------
Combine both half reactions, 2MnO4- + 6I- + 4H2O 2MnO2 + 3I2 + 8OH-
----------------------------------------------------------------

Atomic Structure & Basics of Chemistry Page 62


Finally, checked the coefficient of elements shows that the equation is balanced in terms of
both atoms and charges.

Example.2 Balance the following redox in acidic medium by ion-electron method


H2S + HNO3 H2SO4 + NO2 + H2O
Write the ionic equation for the above reaction,
H2S + H+ + NO3- 2H+ + SO42- + H2O
Split into two half reactions, H2S H2SO4 (oxidation)
(-2) (+6)
-
NO3 NO2 (reduction)
(+5) (+4)
Balance the ‘O’ and ‘H’ atoms in the above two half reactions,
H2S + 4H2O H2SO4 + 8H+
NO3- + 2H+ NO2 + H2O
Balance the charges on both half reactions,
H2S + 4H2O H2SO4 + 8H+ + 8e-
NO3- + 2H+ + e- NO2 + H2O
Compare the both half reactions and equalize their charges, so, multiply the reduction
equation by 8, and if any common ions, molecules and charges are there cancel it, we get,
H2S + 4H2O H2SO4 + 8H+ + 8e-
8NO3- + 16H+ + 8e- 8NO2 + 8H2O
------------------------------------------------------------
H2S + 8HNO3 H2SO4 + 8NO2 + 4H2O
----------------------------------------------------------

Atomic Structure & Basics of Chemistry Page 63

View publication stats

Вам также может понравиться